Sie sind auf Seite 1von 105

P U P

Instructional Materials in
GEED 10053
Mathematics in the Modern World

compiled by

DMS Faculty

College of Science
Polytechnic University of the Philippines

2020
for the sole noncommercial use of the
Faculty of the Department of Mathematics and Statistics
Polytechnic University of the Philippines

2020

Conributors:

Abdul, Alsafat
Atienza, Jacky Boy
Bang-as, Pamela
Bernardino, Rhea
Cabanig, Sarah Jean
Criseno, Regine
Dilla, Perlyn Mae
Duarte, Rafael
Elizon, Katrina
Equiza, Cynthia
Hernandez, Andrew
Isaac, Emelita
Lara, Jose Alejandro Constantino
Longhas, Paul Ryan
Macatangay, Shaina Lyra
Malvar, Rolan
Nuguid, Kenneth James
Saguindan, Ian
Sta. Maria, John Patrick
Republic of the Philippines
POLYTECHNIC UNIVERSITY OF THE PHILIPPINES
COLLEGE OF SCIENCE
Department of Mathematics and Statistics

Course Title : MATHEMATICS IN THE MODERN WORLD

Course Code : GEED 10053

Course Credit : 3 units

Pre-Requisite : GENERAL MATHEMATICS, STATISTICS AND PROBABILITY (SHS)

Course Description :

The course deals with the nature of mathematics, appreciation of its practical, intellectual and
aesthetic dimensions, and application of mathematical tools in daily life. It also bridges the study of
mathematics to other domains of interest like business, finance, social sciences and arts and design.

COURSE LEARNING PLAN

Week Dates Topics and Subtopics

I. Nature of Mathematics

Mathematics in Nature

Week 1 10/5 – 10/11 1. Patterns and Numbers in Nature


2. Fibonacci Sequence
3. Mathematics for Our World

Language of Mathematics
10/12 – 10/18
Week 2-3
10/19 – 10/25 1. Propositions and Logical Connectives
2. Sets, Operations and Venn Diagrams

Problem Solving
10/26 – 11/1
Week 4-5 1. Inductive and Deductive Reasoning
11/2 – 11/8
2. Polya’s Guidelines for Problem Solving
3. Mathematical Problems involving Patterns

II. Mathematics as a Tool: Statistics and Data Management

Week 6 11/9 – 11/15 Data Gatheing and Sampling Techniques


1. Steps in Statistical Investigation
2. Sampling Techniques, Sample Size Considerations, Methods of
Data Collection
3. Levels of Measurement

Data Presentation

Week 7 11/16 – 11/22 1. Tabular Presentations: Frequency Distributions and


Crosstabulations
2. Graphical Presentations: Graphs, Charts, Time Series Plots

Descriptive Measures

Week 8 11/23 – 11/29 1. Measures of Central Tendency


2. Measures of Dispersion or Variation

III. Special Topics

Graphical Solution of LP Models

Week 9-11 11/30– 12/20 1. Modeling with Linear Programming


2. Solution Set of Systems of Linear Inequalities in Two Variables
3. Graphical Solution for a Linear Programming Model

Mathematics of Graphs

1. Basic Concepts in Graphs and Networks


Week 12-14 1/4– 1/24 2. Euler’s Theorem and Fleury’s Algorithms
3. Hamiltonian Circuits and the Traveling Salesman Problem
4. Spanning Trees and Kruskal Algorithm
5. Graph Coloring

FINAL ASSESSMENT

*Note: Graphical Solution of LP Models and Mathematics of Graphs and Networks are required special topics for
the programs under the ff. colleges: Architecture and Fine Arts (CAFA), Computer and Information Sciences
(CCIS), Engineering (CE), Science (CS) and Technology (ITECH).

COURSE GRADING SYSTEM

The final grade will be based on the weighted average of the student’s scores on each test assigned at
the end of each lesson. The final SIS grade equivalent will be based on the following table according to
the approved University Student Handbook.

Class Standing (CS) = (Weighted Average of all the Chapter/Unit Tests ÷ 2) + 50%
Midterm and/or Final Exam (MFE) = (Weighted Average of the Midterm and/or FinalTests ÷ 2) + 50%

Final Grade = (70% x CS) + (30% x MFE)


SIS Grade Percentage/Equivalent Description
1.00 97.00 - 100 Excellent
1.25 94.00-96.99 Excellent
1.50 91.00-93.99 Very Good
1.75 88.00-90.99 Very Good
2.00 85.00-87.99 Good
2.25 82.00-84.99 Good
2.50 79.00-81.99 Satisfactory
2.75 77.00-78.99 Satisfactory
3.00 75.00-76.99 Passing
5.00 65.00-74.99 Failure
Inc Incomplete
W Withdrawn
Final grades are rounded off to 2 decimal places.

Reference Materials:

• Smith, Karl J. The Nature of Mathematics. 12ed. Cengage Learning. 2012


• Angel, Abbott, Runde. Survey of Mathematics with Applications. 10ed. Pearson. 2016
• Lippman, David. Mathematics in Society. 2ed. 2017
• Thomas, Christopher. Schaum’s Outline of Mathematics for the Liberal Arts. McGrawHill. 2009

Prepared by: Noted by:

Kenneth James T. Nuguid/ Ian J. Saguindan Edcon B. Baccay


Faculty Members Chairperson
Department of Mathematics and Statistics Department of Mathematics and Statistics

Approved by:

Dr. Lincoln A. Bautista


Dean, College of Science

Dr. Emanuel C. de Guzman


Vice President for Academic Affairs
Contents

S
DM
1 Mathematics in Our World . . . . . . . . . . . . . . . . . . . . . . . . . . . . . . . . . 5
1.1 Overview: What is mathematics? . . . . . . . . . . . . . . . . . . . . . . . . . 5
1.2 Patterns and Numbers in Nature . . . . . . . . . . . . . . . . . . . . . . . . . 5
1.3 Fibonacci Sequence . . . . . . . . . . . . . . . . . . . . . . . . . . . . . . . . 12
1.4 Mathematics for Our World . . . . . . . . . . . . . . . . . . . . . . . . . . . . 17
2 Logic and Sets . . . . . . . . . . . . . . . . . . . . . . . . . . . . . . . . . . . . . . . 22
2.1 Propositions . . . . . . . . . . . . . . . . . . . . . . . . . . . . . . . . . . . . 22
2.2 Compound Propositions . . . . . . . . . . . . . . . . . . . . . . . . . . . . . . 23
2.3 Sets . . . . . . . . . . . . . . . . . . . . . . . . . . . . . . . . . . . . . . . . 28
3 Problem Solving . . . . . . . . . . . . . . . . . . . . . . . . . . . . . . . . . . . . . . 34
P
3.1 Inductive and Deductive Reasoning . . . . . . . . . . . . . . . . . . . . . . . . 34
3.2 George Polya’s Guidelines for Problem Solving . . . . . . . . . . . . . . . . . . 37
4 Statistics and Data Management . . . . . . . . . . . . . . . . . . . . . . . . . . . . . 41
PU

4.1 Basic Concepts . . . . . . . . . . . . . . . . . . . . . . . . . . . . . . . . . . . 41


4.2 Steps in Statistical Investigation . . . . . . . . . . . . . . . . . . . . . . . . . . 45
4.3 Sampling and Sampling Techniques . . . . . . . . . . . . . . . . . . . . . . . . 45
4.4 Sample Size Considerations . . . . . . . . . . . . . . . . . . . . . . . . . . . . 46
4.5 Methods of Data Collection . . . . . . . . . . . . . . . . . . . . . . . . . . . . 49
4.6 Levels of Measurement . . . . . . . . . . . . . . . . . . . . . . . . . . . . . . . 49
4.7 Presentation of Data . . . . . . . . . . . . . . . . . . . . . . . . . . . . . . . . 50
4.8 Measures of Central Tendency . . . . . . . . . . . . . . . . . . . . . . . . . . . 54
4.9 Measures of Dispersion or Variability . . . . . . . . . . . . . . . . . . . . . . . 58
5 Linear Programming . . . . . . . . . . . . . . . . . . . . . . . . . . . . . . . . . . . . 64
5.1 Modeling with Linear Programming . . . . . . . . . . . . . . . . . . . . . . . . 64
5.2 Solution Set of Systems of Linear Inequalities in Two Variables . . . . . . . . . 68
5.3 Graphical Solution for a Linear Programming Model . . . . . . . . . . . . . . . 69
Lesson 0 4

6 Mathematics of Graphs . . . . . . . . . . . . . . . . . . . . . . . . . . . . . . . . . . 73
6.1 Graph Concepts and Models . . . . . . . . . . . . . . . . . . . . . . . . . . . . 73
6.2 Euler’s Theorems and Fleury’s Algorithms . . . . . . . . . . . . . . . . . . . . . 77
6.3 Hamilton Circuits, Hamilton Paths and the
Traveling-Salesman Problems . . . . . . . . . . . . . . . . . . . . . . . . . . . 83
6.4 Spanning Trees and Kruskal’s Algorithm . . . . . . . . . . . . . . . . . . . . . 90
6.5 Graph Coloring . . . . . . . . . . . . . . . . . . . . . . . . . . . . . . . . . . . 92

S
DM
P
PU

All Rights Reserved. 2020 Abdul, Atienza, et. al.


Lesson 1 5

Lesson 1: Mathematics in Our World

Learning Outcomes
At the end of the lesson, the students are able to:

1. identify patterns in nature in the world;

2. articulate the importance of mathematics in one’s life;

3. argue about the nature of mathematics, what it is how it is expressed, represented and used;

S
4. enumerate and discuss the role of mathematics in various disciplines;

5. express appreciation for mathematics as a human endeavor.

1.1

DM
Overview: What is mathematics?
Mathematics can be defined in many ways. For some people, Mathematics is just the study of numbers.
For others, it is a set of problem-solving tools, a language, a process of thinking, and a study of patterns
among others. Whatever point of view is taken, there is no denying the reality that mathematics is
everywhere. Individuals from around the world use math in their daily lives. Mathematics has various
applications in the world. However, Mathematics is not only concerned with everyday problems, but also
with using imagination, intuition and reasoning to find new ideas and to solve puzzling problems. Math-
P
ematics is a branch of science, which deals with numbers and their operations. It involves calculation,
computation, solving of problems etc. Its dictionary meaning states that, ‘Mathematics is the science
of numbers and space’ or ‘Mathematics is the science of measurement, quantity and magnitude.’ It is
exact, precise, systematic and a logical subject.
PU

Mathematics helps us to organize and systemize our ideas about patterns; in so doing, not only can we
admire and enjoy these patterns, we can also use them to infer some of the underlying principles that
govern the world of nature.

In this lesson, attention will be focused on the nature of mathematics, patterns and numbers in nature
and the world and the uses of mathematics.

1.2 Patterns and Numbers in Nature


What are patterns anyway? We usually think of it as anything that repeats again and again. A pattern is
an arrangement which helps observers anticipate what they might see or what happens next. A pattern
also shows what may have come before. A pattern organizes information so that it becomes more useful.

All Rights Reserved. 2020 Abdul, Atienza, et. al.


Lesson 1 6

The human mind is programmed to make sense of data or to bring order where there is disorder. It seeks
to discover relationships and connections between seemingly unrelated bits of information. In doing so,
it sees patterns.

According to the National Council of Teachers of Mathematics


(1991) defines the nature of mathematics as follows: Mathe-
matics is a study of patterns and relationship, a way of thinking,
an art, a language, and a tool. It is about patterns and rela-
tionships. Numbers are just a way to express those patterns

S
and relationships. patterns

DM Patterns are everywhere. They are deeply embedded all around


us. You can observe patterns- things like colors, shapes, ac-
tions, line or curves of building, pathways or even in the gro-
cery store where boxes of various items are lined up. Number
patterns such as 2,4,6,8 and 5,10,15,20 are among the first
patterns encountered in younger years.

As we advance, we encounter more patterns and discover that number patterns are not restricted to a few
types. They could be ascending, descending, multiples of a certain number. We learned patterns through
P
the concept of functions and sequences like arithmetic and geometric sequences. Number patterns, logic
patterns, geometric patterns and word patterns are examples of the various patterns we learned in school.
However, patterns are not limited to these types. One can observe patterns in nature, art, architecture,
PU

human behavior, anywhere. On this section, we will discuss the different patterns in nature, arts and
architecture.

Patterns in nature are visible regularities of form found in the natural world. These patterns recur in
different contexts and can sometimes be modeled mathematically. Natural patterns include symmetries,
fractals, spirals, meanders, waves, foams, tessellations, cracks, and stripes. Studying patterns allows one
to watch, guess, create, and discover. The present mathematics is considerably more than arithmetic,
algebra, and geometry. The method of doing it has advanced from simply performing computations or
derivations into observing patterns, testing guesses, and evaluating results.

Let us focus on the different types of symmetric patterns, analyze and observe the similarities as well
as the differences and give examples of these types of patterns as seen in nature, arts, architecture and
mathematics.

All Rights Reserved. 2020 Abdul, Atienza, et. al.


Lesson 1 7

Symmetry

When we think of patterns, we usually think of it as something that repeats again and again. The math
of symmetry can describe what this repetition may look like and as well as why some objects seem more
orderly and organized than others. That is why we can say symmetry is the fundamental “language” of
patterns.

What is symmetry? Can you give examples of objects that are symmetric? Why do you consider them
symmetric? Is it because of balanced proportions? Or is it because you can rotate, translate or reflect
and they still look the same?

S
DM
Symmetry can be found everywhere. It can be seen from different viewpoints namely; nature, the arts and
P
architecture, mathematics; especially geometry and science. Symmetry occurs when there is congruence
in dimensions, due proportions and arrangement. It provides a sense of harmony and balance. In fact,
symmetry is one of the foremost predominant themes in arts, design and architecture all over the world
and throughout human history. Mathematical symmetry can also be explained as the passage of time, a
PU

spatial relationship and an aesthetic element found within abstract objects, theoretic models, language,
music and even knowledge itself.

Reflection or Bilateral Symmetry

Bilateral or reflection symmetry is the simplest kind of symmetry. It is one of the most common
kinds of symmetry that we see in the natural world. It can also be called mirror symmetry because an
object with this symmetry looks unchanged if a mirror passes through its middle. In other words, the
objects have a left side and a right side that are mirror images of each other. If a shape can be folded
in half so that one half fits exactly on top of the other, then we say that the shapes are symmetric. The
fold is called a line of symmetry because it divides the shape into two equal parts. Bilateral-symmetric
objects have at least one line or axis of symmetry. The lines of symmetry may be in any direction.

All Rights Reserved. 2020 Abdul, Atienza, et. al.


Lesson 1 8

S
Radial Symmetry
DM
Images Exhibiting Bilateral Symmetry

Radial symmetry is rotational symmetry around a fixed point known as the center. Images with more than
one lines of symmetry meeting at a common point exhibits a radial symmetry. An equilateral triangle
and circles are examples. You can cut along three different axes on the equilateral triangle while a circle
P
can be cut along an infinite number of axes. Consider the photo below. It has rotational symmetry. How
many lines of symmetry are possible?
PU

Radial symmetry can be found both in natural and human made objects. The photos below are examples
of rotational symmetry that can be found in the world around us.

All Rights Reserved. 2020 Abdul, Atienza, et. al.


Lesson 1 9

S
Did you know that there are other classifications of symmetric patterns. Patterns in the plane are usually
divided into three groups. These are rosette patterns (those that repeat in no direction), frieze patterns

overlaps, like wallpaper patterns.

Rosette Patterns
DM
( those that repeat in exactly one direction) and wallpaper patterns (those that repeat in more than
one direction). Let us define, discuss and identify examples of these patterns from nature and the arts.
Included in the discussion is what we call tessellations which completely cover a plane without gaps or

Rosette patterns consist of taking motif or an element and rotating and/or reflecting that element.
There are two types of rosette patterns namely cyclic and dihedral. A rosette pattern is cyclic if it
only admits rotational symmetries. On the other hand, a rosette pattern is dihedral if it admits both
P
rotational symmetries and bilateral or reflectional symmetries. The figures below exhibit rosette patterns.
Can you identify which of them are cyclic? dihedral?
PU

Frieze Patterns

A frieze or border pattern is a pattern in which a basic motif repeats itself over and over in one
direction. It extends to the left and right in a way that the pattern can be mapped onto itself by a

All Rights Reserved. 2020 Abdul, Atienza, et. al.


Lesson 1 10

horizontal translation. We can usually find these patterns in unique places like on the walls of buildings,
fabrics, borders of rugs and tiled floor.

Mathematicians have already classified all the different types of frieze patterns. It turns out that there
are only seven types.

1. Hop. The frieze pattern only admits a translational symmetry.

S
2. Step. The frieze pattern only admits a translational and glide symmetries.

DM
3. Sidle. The frieze pattern only admits translations and vertical reflections.

4. Spinning Hop. The frieze pattern only admits translations and 180◦ rotations (half-turns).
P
PU

5. Spinning Siddle. The frieze pattern only admits translations, vertical reflections, rotations, and
glide reflections.

All Rights Reserved. 2020 Abdul, Atienza, et. al.


Lesson 1 11

6. Jump. The frieze pattern only admits translations, a horizontal reflection, and glide reflection.

7. Spinning Jump. The frieze pattern admits translations, vertical reflections, horizontal reflections,
rotations, and glide reflections.

S
Mathematician John B. Conway invented the names of these frieze patterns.

Wallpaper Patterns

DM
A wallpaper pattern is a pattern with translation symmetry in two directions. It is, therefore, essentially
an arrangement of friezes stacked upon one another to fill the entire plane. Any particular wallpaper
pattern is made up of a combination of the following symmetries; reflection, rotation and glide reflection.
According to Nocon (2016), in order for a plane figure to be considered a wallpaper pattern, it must
have at least the basic unit, one copy by translation, and a copy of these two by translation in the second
direction. There must be at least two rows, each one of at least two units long.
P
PU

Beautiful patterns can be created by repeating geometric and artistic motifs according to the symmetry
of the wallpaper groups, as exemplified in works by M. C. Escher and in the patterns created by I. Bakshee

All Rights Reserved. 2020 Abdul, Atienza, et. al.


Lesson 1 12

in the Wolfram Language using Artlandia, illustrated above. There are 17 different wallpaper patterns.

Using intricate techniques, mathematicians were able to classify every wallpaper patterns possible. It is
shown that there are only 17 distinct types of wallpaper patterns.

S
Some Wallpaper Patterns

Tesselations

DM
A tessellation or tiling is a repeating pattern of figures that covers a plane with no gaps or overlaps.
It is just like a wallpaper group in which patterns are created by repeating a shape to fill the plane.

Tessellations can be created with translations, rotations, and reflections. Tessellations can be seen in
nature, arts and everyday life. Pavements, snake skin, turtle shell and a honeycomb are just few of many
examples of tessellation we see around us. A honeycomb is a perfect example of a natural tessellation.
It uses regular hexagons to form this natural mosaic around the surface area of the hive. Since these are
P
regular hexagons, each interior angle of each hexagon are 120 degrees, and all the angles in one of the
hexagons equal 720 degrees.
PU

Examples of Tesselations

1.3 Fibonacci Sequence


We start with 1 and another 1. Add them, we get 2. Add 1 and 2, we get 3. Add 2 and 3, we get 5.
Add 3 and 5, we get 8. If we continue repeating the process, we obtain the sequence

1; 1; 2; 3; 5; 8; 13; : : :

All Rights Reserved. 2020 Abdul, Atienza, et. al.


Lesson 1 13

which is known as the Fibonacci sequence. The Fibonacci sequence was invented by the Italian
Leonardo Pisano Bigollo (1180-1250), who is known in mathematical history by several names: Leonardo
of Pisa (Pisano means “from Pisa”) and Fibonacci (which means “son of Bonacci”).
To formally, define the Fibonacci sequence, we start by defining F1 = 1 and F2 = 1. For n > 2, we
define
Fn := Fn−1 + Fn−2 :

The sequence F1 ; F2 ; F3 ; : : : is then the Fibonacci sequence. Such a definition is called a recursive
definition because it starts by defining some initial values and defines the next term as a function of
the previous terms.

S
If we take the ratio of Fn to Fn−1 for n ≥ 1,

n
1
2
3
4
5
6
Fn
1
1
2
3
5
8
DM
Fn =Fn−1
-
1
2
1.5
1:666 : : :
1.6
n
8
9
10
11
12
13
Fn
21
34
55
89
144
233
Fn =Fn−1
1:61538 : : :
1:61904 : : :
1:61764 : : :
1:61818 : : :
1:61797 : : :
1:61805 : : :
7 13 1.625 14 377 1:618025 : : :
P
we see that as n gets larger and larger, the ratio gets closer and closer to a value denoted by ’. The
number ’ is called as the golden ratio and can be formally defined as
PU

Fn
’ := lim :
n−→∞ Fn−1

The symbol lim means ‘the limit as n approaches infinity’ which is usually studied in a calculus course.
n−→∞
It can be calculated that the exact value of ’ is

1+ 5
’= ≈ 1:6180339887 : : : :
2

1− 5
If we denote by ’ := , we can write the nth Fibonacci number explicitly using the formula
2
’n − ’n
Fn = √ :
5

This is known as the Binet Formula.

All Rights Reserved. 2020 Abdul, Atienza, et. al.


Lesson 1 14

2
3

1 1

S
DM
Do you see the Fibonacci Numbers? The red curve is known as the Fibonacci Spiral.

A rectangle whose side ratio (length:width) equals ’ is called a golden rectangle.

George Dvorsky (2013) emphasized that the Fibonacci sequence has captivated mathematicians, scien-
tists, artists and designers for centuries. It is a sequence with many interesting properties. Among these
is its visibility in nature. Most, if not all, natureâĂŹs most beautiful patterns contain Fibonacci numbers.

The Fibonacci numbers appear in nature in various places. These numbers are evident at the flower
P
head of a sunflower or daisy. Spirals are also easier to see and to count on pineapples and pine cones.
Fibonacci numbers are there on broccoli florets and flowers and on the arrangement of leaves around
stems on many plants too.
PU

• Pinecones, Speed Heads, Vegetables and Fruits


Spiral patterns curving from left and right can be seen at the array of seeds in the center of a
sunflower. The sum of these spirals when counted will be a Fibonacci number. You will get two
consecutive Fibonacci numbers if you divide the spirals into those pointed left and right. The
seed pods on a pinecone are also arranged in a spiral pattern. Each cone consists of a pair of
spirals, each one spiraling upwards in opposing directions. Spiral patterns can also be deciphered
in cauliflower and pineapples. Fibonacci sequence appears on these fruits and vegetables.

• Flowers and Branches


Most flowers express the Fibonacci sequence if you count the number of petals on these flowers.
For example, lilies and irises have three petals, roses and buttercups have five, delphiniums have
eight petals and so on. Some plants also exhibit the Fibonacci sequence in their growth points, on
the places where tree branches form or split. A trunk grows until it produces a branch, resulting

All Rights Reserved. 2020 Abdul, Atienza, et. al.


Lesson 1 15

in two growth points. The main trunk then produces another branch, resulting in three growth
points and then the trunk and the first branch produce two more growth points, bringing the total
to five as illustrated on the image below.

S
• Honeybees

DM
The family tree of a honey bee perfectly resembles the Fibonacci sequence. A honeybee colony
consists of a queen, a few drones and lots of workers. The following image below shows how the
family tree relates.
P
PU

• The Human Body


The human body has many elements that show the Fibonacci numbers and the golden ratio. Most
of your body parts follow the Fibonacci sequence and the proportions and measurements of the
human body can also be divided up in terms of the golden ratio.

• Geography, Weather and Galaxies Fibonacci numbers and the relationships between these
numbers are evident in spiral galaxies, sea wave curves and in the patterns of stream and drainages.
Weather patterns, such as hurricanes and whirlpools sometimes closely resemble the Golden Spiral.
The milky way galaxy and some other galaxies have spiral patterns. Planets of our solar system
and their orbital periods are closely related to the golden ratio.

All Rights Reserved. 2020 Abdul, Atienza, et. al.


Lesson 1 16

S
DM
The Golden Ratio and/or the Golden Spiral can also be observed in music, art, and designs. Appearing
in many architectural structures, the presence of the golden ratio provided a sense of balance and
equilibrium. Let’s take a look at a couple of examples.

• Architecture. The Great Pyramid of Giza: The Great Pyramid of Giza built around 2560 BC is
one of the earliest examples of the use of the golden ratio. The length of each side of the base
P
is 756 feet, and the height is 481 feet. So, we can find that the ratio of the vase to height is
756=481 = 1:5717:
PU

The Greek sculptor Phidias sculpted many things including the bands of sculpture that run above
the columns of the Parthenon. Other architectural structures that exhibits the Golden ratio include
the ff: Porch of Maidens, Acropolis, Athens; Chartres Cathedral; and Le Corbussier. Can you name
other structures that has the Golden Ratio?

• Arts. Mona-Lisa by Leonardo Da Vinci: It is believed that Leonardo, as a mathematician tried


to incorporate of mathematics into art. This painting seems to be made purposefully line up with

All Rights Reserved. 2020 Abdul, Atienza, et. al.


Lesson 1 17

golden rectangle.

An Old man by Leonardo Da Vinci: Leonardo Da Vinci explored


the human body involving in the ratios of the lengths of various
body parts. He called this ratio the "divine proportion" and
featured it in many of his paintings. We also have the The
Vetruvian Man (“The Man in Action”) by Leonardo Da Vinci;
Holy Family by Micahelangelo; Crucifixion by Raphael; The
sacrament of the Last Supper by Salvador Dali (1904-1989),
and many more.

S
1.4 Mathematics for Our World

DM
Mathematics is everywhere; whether it is on land, sea or air, online or on the front line, mathematics
underpins every nook and cranny of modern life. Far from a quaint subject to be forgotten upon leaving
school, it is the glue that holds our world.

Roger Bacon (1214-1294), an English Franciscan friar, philosopher, scientist and scholar of the 13th
century, once stated: “Neglect of mathematics works injury to all knowledge, since he who is ignorant
of it cannot know the other sciences or the things of the world.”

Math helps us understand or make sense of the world - and we use the world to understand math. It is
P
therefore important that we learn math contents needed to solve complex problems in a complex world;
learn the mathematical knowledge and skills we need to understand the world and make contributions
to the global community.
PU

Applications of Mathematics in Our World

Mathematics has so many uses of applications.

• Mathematics helps organize patterns and regularities in the world;

• Mathematics helps predict the behavior of nature and many phenomena;

• Mathematics helps control nature and occurrences in the world for our own good;

• Mathematics has applications in many human endeavors.

All Rights Reserved. 2020 Abdul, Atienza, et. al.


Lesson 1 18

S
DM
Mathematics helps organize patterns and regularities in the World

According to Ian Stewart (1995), we live in a universe of patterns. Human mind and culture have de-
veloped a formal system of thought for recognizing, classifying and understanding patterns. This formal
system of thought is what we know now as mathematics. We use mathematics to organize and system-
atize our thoughts and ideas about patterns and other regularities in this world. The development of
P
these new mathematical theories helped paved the way to the thorough understanding of the different
patterns in nature. Stewart (1995) also mentioned that our newfound understanding of natural order and
nature’s secret regularities is being used to steer artificial satellites to new destinations with far less fuel
PU

than anybody had thought possible, to help avoid wear on the wheels of locomotives and other rolling
stock, to improve the effectiveness of heart pacemakers, to manage forests and fisheries, even to make
more efficient dishwashers. But most important of all, it is giving us a deeper vision of the universe in
which we live, and of our own place in it. Yes, mathematics has indeed helped organize patterns and
consistencies in the world.

Mathematics helps predict the behavior of nature and many phenomena.

Mathematics is used to explain why the Sun set, where it went and why it returned because it was easier
to count these events in numbers than to put them into words. Based on historical patterns, we can make
forecasts or predictions to help us prepare for our daily activities.Formulas and other mathematical meth-
ods became a way of using numbers to show how things in nature happen, where and when it will happen.

All Rights Reserved. 2020 Abdul, Atienza, et. al.


Lesson 1 19

Earth scientists have relied in the past on statistical methods to forecast natural hazard events. However,
Benoit Mandelbrot, a professor of mathematical sciences at Yale University described how he has been
using fractals to find order within complex systems in nature, such as the natural shape of a coastline.
As a result of his research, earth scientists are taking Mandelbrot’s fractal approach one step further and
are measuring past events and making probability forecasts about the size, location, and timing of future
natural disasters.
Mathematics helps control nature and occurrences in the world for our own good.

Mathematical modelling and control theory can be used. By mathematical modeling we see the inputs
to events and their most likely outcomes. Knowing these inputs and seeing their consequences and

S
establishing their relationship defined quantitatively, we can prepare for calamities or natural disasters,
or better yet, we can probably stop them from happening.

DM
Control theory is defined as a field of applied mathematics that is relevant to the control of certain phys-
ical processes and systems. As long as human culture has existed, control has meant some kind of power
over the environment and control theory may be viewed as the science of modifying that environment,
in the physical, biological, or even social sense. Control theory played a major role in many technological
advances in the second half of the 20th century.

Mathematics has applications in many human endeavors making it indispensable.

Mathematics existed since the beginning of time, written or unwritten. Its unwritten history is carved in
P
all things found in cosmos , found in the patterns created in nature, appreciated in the juxtaposition of
the heavens and the earth, contrast between darkness and light , made sense in the harmony created not
just by a well-known orchestra but even by the rain drops falling on offshore wind-turbines. Its language,
PU

though considered by many as abstract is in fact easy to grasp when the logic and formula that govern it
are understood by the inquisitive minds of students, bakers, chemists , carpenters and appreciated by the
receptive hearts of the musicians - drummers, guitarists, pianists and composers; dance choreographers,
gymnasts and marathon runners.

Mathematics permeates every area of man’s life , leaving every man convinced of its value. As a tool,
mathematics is indispensable. It is needed by all people in honing their logical thinking and reasoning,
in making wise financial decisions - in budgeting or making both ends meet when financial resources are
scarce. It is needed in choosing the best interior and outdoor designs of houses , offices and business
sites. It is useful in determining traveling time and calculating the amount of fuel needed to get to the
destination. It is not just needed in the classrooms but also at home when doing the mundane baking
or preparing foods for breakfast , dinner or lunch; calculating steps when performing simple to complex
acrobatic stance; determining speed in a short distance or marathon run, preparing chemical solutions in

All Rights Reserved. 2020 Abdul, Atienza, et. al.


Lesson 1 20

a biological or chemical laboratory and the like. Indeed, its application and use are uncountable and the
list of uses it offers is unending.

As it is valuable and integral in the life of man, mathematics as a discipline that Introduces students
with the wide array of possibilities from honing problem-solving skills to enriching aesthetic judgment.

S
DM
P
PU

All Rights Reserved. 2020 Abdul, Atienza, et. al.


Lesson 2 21

Assessment

I. Patterns and Numbers in Nature

(1) Give five examples each of nature having reflection symmetry and radial symmetry.

(2) Compare and contrast (a) rotation and reflection; (b) translation and rotation.

(3) Which upper case letters of the English alphabet look the same after being rotated 90◦ ?
180◦ ?

S
(4) Classify the following frieze patterns based on Conway’s classification.

(a)
DM (c)

(d)

(b) (e)
P
II. Fibonacci Sequence
PU

(1) Enumerate the first twenty Fibonacci numbers.

(2) Use F40 = 63; 245; 986 and F38 = 39; 088; 169 to find the value of F39 . Show your reasoning.

(3) Using the Binet’s formula, calculate F4 .

III. Beyond the Walls (Performance Task)


Look for patterns Inside or outside of your house then take pictures of the patterns explored using
smart phones or digital camera. Explore, take photos, make list and identify what patterns can be
seen in nature inside your house, at the garden or park nearby or any part of the neighborhood.
Showcase your drawing skills by creating original paintings or pictures, poster, photo collage or vlogs
of the different patterns in nature, Fibonacci, golden ratio or the like that you have encountered
on your walk.

All Rights Reserved. 2020 Abdul, Atienza, et. al.


Lesson 2 22

Lesson 2: Logic and Sets

Learning Outcomes
At the end of the lesson, the students are able to

1. identify which are propositions and which are not;

2. construct compound propositions using logical connectives;

3. construct truth tables for propositions;

S
4. test validity of arguments

2.1 Propositions

DM
Mathematics is a language. As in any other types of language, we use sentences to communicate thoughts
and ideas. Mathematics is not an exception. We use propositions to communicate mathematical ideas
precisely.

Definition 1
A proposition is a declarative sentence that can be objectively identified as either true or false,
but not both. If a proposition is true, then its truth value is true and is denoted by T or 1;
otherwise, its truth value is false and is denoted by F or 0.
P
Example 1. Consider the following sentences.
PU

(1) Douglas MacArthur arrived in the Philippines in 1521. (5) Is that your laptop?

(2) Are you insane? (6) Basketball players are handsome.



(3) 2 is an irrational number. (7) There is life in other planets.

(4) Find all x such that xe−x = 2. (8) Welcome to the Philippines!

Immediately, we find that sentences (2), (4), (5), and (8) are not declarative sentences, so they are not
propositions.

Sentence (1) is a proposition because Douglas MacArthur either arrived in the Philippines in 1521 or
not. In fact, this proposition is false because historical records shows that Douglas MacArthur arrived in
the Philippines some time in 1900s.

All Rights Reserved. 2020 Abdul, Atienza, et. al.


Lesson 2 23

Sentence (3) is clearly a true proposition. Although statement (6) is a declarative sentence, it cannot
be considered a proposition because the meaning of the word “handsome” is subjective in nature. Unless
we could agree on an objective definition of “handsome”, then statement (6) cannot be considered a
proposition.

Finally, statement (7) is a proposition. Whether there is life or not in other planets, it doesn’t really
matter. The fact that this sentence is either true or false, and cannot be both true and false, makes it a
proposition. For this example, we still don’t have enough evidence to claim that proposition (7) is true
yet, and we don’t have a proof that it is false either. Hence, only time will tell when can we assign a
truth value for (7), but certainly, it has a truth value.

S
Symbolically, we denote propositions in this lesson using lower case letters, such as p; q; r; s; etc.

Definition 2

DM
The negation of a proposition p is the proposition which is false when p is true; and true when
p is false. The negation of p is denoted by ¬ p.

In the English language, we can simply state the negation of a proposition p by saying “It is not the case
that p.” However, there are many ways to express negations of statements grammatically by replacing
“is/are” by “is not/are not”, etc.

Example 2. Given the statements


P
p : Everyone in Visayas speaks Cebuano.
q : Today is Wednesday.
PU

The corresponding negations are

¬ p : Not everyone in Visayas speaks Cebuano.


¬ q : Today is not Wednesday.

2.2 Compound Propositions


A simple proposition is a proposition with only one subject and only one predicate. For example, the
proposition “Every cat that barks has a PhD.” is a simple proposition. The subject of this proposition
is “every cat that barks” and the predicate is “has a PhD.” In logic, we can combine simple propositions
to form compound propositions using logical connectives. Some of the most common connectives
are “or”, “and”, “but”, “unless”, etc.

All Rights Reserved. 2020 Abdul, Atienza, et. al.


Lesson 2 24

Definition 3
Let p and q be given propositions. The conjunction of p and q is the proposition “p and q”,
denoted by p ∧ q, which is true only when both p and q are true.

In other words, if one of p or q is false, then p ∧ q is false. We summarize this idea using the following
table.

p q p∧q
1 1 1
1 0 0

S
0 1 0
0 0 0

Example 3. Given the propositions


DM
Such a table is called a truth table for p ∧q. The truth table above illustrates the different combinations
of truth values for p and q and the corresponding truth value for the conjunction.

p : 3 is odd.
q : Elephants are mammals.
r : Philippines is a first world country.

We know that p and q are true and r is false. Therefore,


P
p ∧ q : 3 is odd and elephants are mammals.

is true, while
PU

p ∧ r : 3 is odd and Philippines is a first world country.

is false. For a more complicated example, the proposition

(¬ p) ∧ (¬ q) : Neither 3 is odd nor Philippines is a first world country.

is still false, since ¬ p is false.

Definition 4
Let p and q be given propositions. The disjunction of p and q is the proposition “p or q”,
denoted by p ∨ q, which is false only when both p and q are false.

In other words, if one of p or q is true (or both), then p ∨ q is true. The truth table for p ∨ q is given
below.

All Rights Reserved. 2020 Abdul, Atienza, et. al.


Lesson 2 25

p q p∨q
1 1 1
1 0 1
0 1 1
0 0 0

Example 4. Consider the statements p, q and r in the preceding example. The statement

p ∨ q : Either 3 is odd or elephants are mammals.

S
is true. Also,
p ∨ r : Either 3 is odd or Philippines is a first world country:

DM
Example 5. The proposition “Either 3 is odd or there is life in other planets.” is technically true since
the component “3 is odd.” is a true proposition. Whether the proposition “There is life in other planets.”
is true or false, the disjunction is always true.

Example 6. Construct a truth table for the compound statement p ∨ (q ∧ (¬ r )).

Solution. Since each of p, q, and r may assume two distinct truth values, then there are a total of
2 · 2 · 2 = 8 combinations, hence the truth table must contain eight rows as shown below.

p q r ¬ r q ∧ (¬ r ) p ∨ (q ∧ (¬r ))
P
1 1 1 0 0 1
1 1 0 1 1 1
1 0 1 0 0 1
PU

1 0 0 1 0 1
0 1 1 0 0 0
0 1 0 1 1 1
0 0 1 0 0 0
0 0 0 1 0 0

Definition 5
Let p and q be propositions. The conditional statement p −→ q is the proposition “If p, then
q.” is the proposition which is false only when p is true and q is false. The converse, inverse,
and contrapositive of p −→ q are the conditional statements q −→ p, (¬ p) −→ (¬ q), and
(¬ q) −→ (¬ p), respectively.

All Rights Reserved. 2020 Abdul, Atienza, et. al.


Lesson 2 26

The following is the truth table for p −→ q.

p q p −→ q
1 1 1
1 0 0
0 1 1
0 0 1

In the proposition p −→ q, the proposition p is also called as the premise and q is called as the
conclusion. From the truth table, we can see that a conditional statement is trivially true when the

S
premise is false.

Example 7. Suppose that your mother exclaims “If you don’t wash the dishes, then you don’t get

DM
money for a buffet.” In this conditional statement, the premise is “You don’t wash the dishes.” and the
conclusion is “you don’t get money for a buffet.” This statement can only false only when you don’t
wash the dishes but you still get money for the buffet.

Note that there are many ways to say p −→ q aside from “If p, then q.” Alternatively, we can say “q if
p” or “p implies q”, “p is sufficient for q” or “q is necessary for p.”

Example 8. Given the statements p : “ı is irrational.” and q : “3 is less than 2.”, then

p −→ q : If ı is irrational, then 3 is less than 2.


P
the converse of this conditional is

q −→ p : If 3 is less than 2, then ı is irrational.


PU

the inverse is
(¬ p) −→ (¬ q) : If ı is not irrational, then 3 is not less than 2.

and the contrapositive is

(¬ q) −→ (¬ p) : If 3 is not less than 2, then ı is not irrational.

If we assume that p is true and q is false (just like how they really are in mathematics), one verifies that
both p −→ q and (¬ q) −→ (¬ p) are false, while both q −→ p and (¬ p) −→ (¬ q) are true.

We like to emphasize that we write the negation of “ı is irrational” as “ı is not irrational” to emphasize
the fact that we actually don’t assume that the opposite of being irrational is being rational, unless
otherwise stated.

All Rights Reserved. 2020 Abdul, Atienza, et. al.


Lesson 2 27

Definition 6
Let p and q be propositions. The biconditional statement p ↔ q to be read as “p if and only
if q” is the proposition which is true only if both p and q are true or both p and q are false.

p q p↔q
1 1 1
1 0 0
0 1 0
0 0 1

S
Definition 7
A compound proposition is a tautology if its truth value remains true regardless of the truth values

DM
of its component propositions. On the other hand, a compound proposition is a contradiction if
its truth value remains false regardless of the truth values of its component propositions.

Example 9. The compound statement p ∨ (¬ p) is a tautology and the compound statement p ∧ (¬ p)


is a contradiction. This can be observed by looking at the truth table below.

p ¬ p p ∨ (¬ p) p ∧ (¬ p)
1 0 1 0
0 1 1 0
P
Definition 8
Let p and q be propositions (possibly compound). We say that p logically implies q, expressed
PU

as p =⇒ q, if the conditional statement p −→ q is a tautology. If p =⇒ q and q =⇒ p, we


say that p and q are logically equivalent and we write p ⇐⇒ q. A compound proposition that
is neither a tautology nor a contradiction is called a contingency.

Example 10. By constructing truth tables, show that p =⇒ p ∨ q and p ∧ q =⇒ p.

Solution.
p q p ∨ q p ∧ q p −→ (p ∨ q) (p ∧ q) −→ p
1 1 1 1 1 1
1 0 1 0 1 1
0 1 1 0 1 1
0 0 0 0 1 1

All Rights Reserved. 2020 Abdul, Atienza, et. al.


Lesson 2 28

In logic, the implication p =⇒ (p∨q) is called as the law of addition and the implication (p∧q) =⇒ p
is the law of simplification.

The following are some of the most common equivalences in logic.

Theorem 1
Let p; q; and r be propositions.

1. p ⇐⇒ q if and only if p ←→ q is a tautology.

2. p ⇐⇒ p.

S
3. p ∨ q ⇐⇒ q ∨ p and p ∧ q ⇐⇒ q ∧ p. (commutative properties)

properties)

6. De Morgan’s Laws DM
4. p ∨ (q ∨ r ) ⇐⇒ (p ∨ q) ∨ r and p ∧ (q ∧ r ) ⇐⇒ (p ∧ q) ∧ r . (associative properties)

5. p ∨ (q ∧ r ) ⇐⇒ (p ∨ q) ∧ (p ∨ r ) and p ∧ (q ∨ r ) ⇐⇒ (p ∧ q) ∨ (p ∧ r ). (distributive

(a) ¬(p ∨ q) ⇐⇒ (¬ p) ∧ (¬ q).


(b) ¬(p ∧ q) ⇐⇒ (¬ p) ∨ (¬ q)

7. p −→ q ⇐⇒ (¬ p) ∨ q.
P
8. ¬(p −→ q) ⇐⇒ p ∧ (¬ q).

9. p −→ q ⇐⇒ (¬ q) −→ (¬ p).
PU

10. p ←→ q ⇐⇒ (p −→ q) ∧ (q −→ p):

2.3 Sets
One of the basic concepts every student of mathematics must know is that of sets.
Definition 9
A set is a well-defined collection of objects called elements.

A collection is well-defined if for any given object we can objectively decide whether it is or is not in the
collection. Any object which belongs to a given set is said to be an element of or a member of the
given set.

All Rights Reserved. 2020 Abdul, Atienza, et. al.


Lesson 2 29

Example 11.

1. The collection of all letters in the English Alphabet is a set.

2. The collection of all handsome guys is not a set, because one cannot objectively identify if a given
guy is handsome or not, because the word “handsome” is subjective in nature.

Upper case letters are usually used to name sets. A set A can be commonly described in three ways,
by (a) listing (roster) method, (b) by set-builder notation or(c) by descriptive method. The listing

S
method describes the set by listing all the elements between braces and separated by commas (note:
in enumerating the elements of a certain set, each element is listed only once and the arrangement of
elements in the list is immaterial). The set-builder notation uses a variable (a symbol, usually a letter,

DM
that can represent different elements of a set), braces, and a vertical bar | that is read as "such that".
This is usually used when the elements are too many to list down. The descriptive method uses a
short verbal statement to describe the set.

Example 12. Using the roster method, the set of months in a year that ends with letter ‘y’ can be
represented by {January, February, May, July}.

Example 13. The set {2; 3; 4; 5; 6; 7; 8; 9} in set-builder notation is

{x | x is an integer greater than 1 but less than 10}:


P
If a is an element of a set A, we write a ∈ A. Otherwise, we write a ∈
= A. There are sets with no
elements. Such a set is said to be empty and we use the symbol ? to denote the empty set. A set
PU

with only one element is called a unit set or a singleton.

Example 14. The set of integers between 1 and 2 is empty, while the set of even prime numbers is a
singleton.

For future discussion, we will use the following notations:

• N for the set of natural or counting numbers (positive integers): {1; 2; 3; 4; :::}

• Z for the set of integers: {::: − 4; −3; −2; −1; 0; 1; 2; 3; :::}


a
 ff
• Q for the set of rational numbers: | a; b ∈ Z; b 6= 0
b
• R for the set of real numbers

All Rights Reserved. 2020 Abdul, Atienza, et. al.


Lesson 2 30

A set A is said to be finite if it is possible to list down all the elements of A in a list. Otherwise, A is
said to be infinite. If A is finite, the cardinality of A is the number of elements of A, which is denoted
by n(A).

Example 15. The set of all letters in the English Alphabet is finite and its cardinality is 26, because
there are 26 distinct letters in the English alphabet. On the other hand, the set of all even integers in
infinite.

Definition 10
Let A and B be sets. We say that A is a subset of B and write A ⊆ B if every element of A is

S
an element of B. We say that A and B are equal and write A = B if A ⊆ B and B ⊆ A.

Remarks.

1. For any set A, A ⊆ A and ? ⊆ A.

DM
2. If A and B are finite sets and A = B, then n(A) = n(B).

Example 16. Let A be the set of all mathematicians 20 feet high and B be the set of all PUP students.
Then A = ?. By Remark (1) above, A ⊆ B: Therefore, we can conclude that every mathematician 20
feet high is a PUP student.

Two finite sets A and B are said to be equivalent if and only if n(A) = n(B). Note that equal sets are
necessarily equivalent bu equivalent sets need not be equal.
P
Example 17. Let A = {x | x is a prime number less than 20} and B = {1; 2; 3; 4; 5; 6; 7; 8} are equiv-
alent since n(A) = 8 = n(B), however, A 6= B.
PU

Definition 11
Let A and B be sets. The union of A and B is defined as

A ∪ B = {x | x ∈ A or x ∈ B}:

The intersection of A and B is

A ∩ B = {x | x ∈ A and x ∈ B}:

Then relative complement of B in A is the set

A \ B := {x ∈ A | x ∈
= B}:

We could represent A ∪ B, A ∩ B, and A \ B in terms of Venn Diagrams as shown below.

All Rights Reserved. 2020 Abdul, Atienza, et. al.


Lesson 2 31

Example 18. Let A = {0; 1; 3; 5; 7} and B = {1; 2; 4; 7; 9}. Then A ∪ B = {0; 1; 2; 3; 4; 5; 7; 9},

S
A ∩ B = {1; 7} and A \ B = {0; 3; 5}.

DM
In most of the interesting instances in mathematics, we normally talk about a particular set of objects
at a given time. The set of all objects of interest is called as the universal set, generically denoted as
U . If A ⊂ U , the complement of A is defined as the set

A0 = U \ A = {x ∈ U ∈ x ∈
= A}:

Using the De Morgan’s Law of logic, one can readily verify that

(A ∪ B)0 = A0 ∩ B 0 and (A ∩ B)0 = A0 ∪ B 0 :


P
We can use our knowledge of sets to solve some word problems.
PU

Example 19. At a certain high school, each student is a member of the English Club, the Science
Club, or the Mathematics Club. Of the 79 students asked, 33 are members of the English Club, 37 are
members of the Math Club, and 37 are members of the of the Science club. Furthermore, 7 are members
of both the English and the Math Clubs, 12 are members of both the English and the Science Clubs,
and 9 are members of the Science and Math Clubs. No high school student is a member of all the three
clubs. How many joined only the Math Club?

Solution Let E, S, and M denote the sets of members of English, Science, and Mathematics Club,
respectively. As given in the problem, the universal set U has cardinality n(U ) = 79, n(E) = 33,
n(M) = 37, and n(S) = 37. Furthermore, n(E ∩ M) = 7, n(E ∩ S) = 12, and n(S ∩ M) = 9. The last
condition imply that E ∩ S ∩ M = ?. This situation can be represented by the following Venn diagram.

All Rights Reserved. 2020 Abdul, Atienza, et. al.


Lesson 2 32

S
Since n(E ∩ S ∩ M) = 0, then the number of students who only joined the Mathematics Club is

DM
n(M) − n(E ∩ M) − n(S ∩ M) = 37 − (7 + 9) = 21:
P
PU

All Rights Reserved. 2020 Abdul, Atienza, et. al.


Lesson 2 33

Assessment
1. Write each statement in words. Let p: The plane is on time. Let q: The sky is clear.

(a) p ∧ (¬ q)
(b) q → (p ∨ ¬p)
(c) p ↔ q

2. Construct a truth table for each proposition.

(a) [(p ∧ q) ∨ r ] ↔ [(p ∧ r ) ∨ (q ∧ r )]

S
(b) [(p ∧ r ) → (q ∧ ¬r )] → [(p ∧ q) ∨ r )]

3. Prove the De Morgan’s Laws by constructing truth tables.

B = {s, q, u, a, r, e };
C = {h, e, x, a, g, o, n, s }
Determine the following:

(a) A ∪ (B ∩ C)
DM
4. Let U := Letters in the English Alphabet = {a, b, c, . . . ,x, y, z}
A = {t, r, i, a, n, g, l, e, s}

(b) (A ∪ B)0 ∩ C
P
(c) (A ∩ C) ∪ (B ∩ C)
(d) A ∩ (C ∩ U )0
(e) n[(A ∪ B) ∩ (B ∪ C)]
PU

5. A survey of 90 customers was taken at Barnes & Noble regarding the types of books purchased.
The survey found that 44 purchased mysteries, 33 purchased science fiction, 29 purchased romance
novels, 13 purchased mysteries and science fiction, 5 purchased science fiction and romance novels,
11 purchased mysteries and romance novels, and 2 purchased all three types of books (mysteries,
science fiction, romance novels). How many of the customers surveyed purchased

(a) mysteries only?


(b) mysteries and science fiction, but not romance novels?
(c) mysteries or science fiction?
(d) romance novels or mysteries, but not science fiction?
(e) exactly two types (mysteries, science fiction, romance novels)?

All Rights Reserved. 2020 Abdul, Atienza, et. al.


Lesson 3 34

Lesson 3: Problem Solving

Learning Outcomes
At the end of the lesson, the students are able to

1. differentiate between inductive and deductive reasoning;

2. utilize inductive reasoning to form conjectures;

3. use deductive reasoning to prove a conjecture;

S
4. state the Polya’s four steps in problem solving;

5. solve mathematical problems using the Polya’s four steps.

3.1

DM
Inductive and Deductive Reasoning
Human beings are said to be rational creatures because we use reasoning to come up with sound decisions
that we have to make everyday. Reasoning is our ability to use logical thinking to come up with a
decision. There are two major types of reasoning: inductive and deductive. We first talk about inductive
reasoning.

Definition 12
P
Inductive Reasoning is the process of reasoning that arrives at a general conclusion based on
the observation of specific examples.

Normally, we use inductive reasoning when we need to come up with a general conclusion, known as
PU

a conjecture, by observing certain events or examples. Generally speaking, our conjectures could be
wrong. Examples which can negate our conjectures are called counterexamples.

Example 20. In the past 30 days, we observed that the sun has risen in the east. Using inductive
reasoning, we may conjecture that the sun will rise in the east tomorrow.

Example 21. Consider the odd numbers 3; 5; 7; and 9. If we take their squares, we see that 32 = 9,
52 = 25, 72 = 49 and 92 = 81. We can observe that the squares of the given odd numbers are all odd
as well. Using inductive reasoning, we may conjecture that the square of an odd integer is also odd.

Testing Conjectures
Logically speaking, we cannot prove a general statement from a number of specific examples unless there
are only finitely many examples and we can exhaust them. However, only one counter example can prove

All Rights Reserved. 2020 Abdul, Atienza, et. al.


Lesson 3 35

that our conjecture is false.

Example 22. Let n be a positive integer. Select n distinct points at random in the circumference of a
circle and connect every pair of points in this collection by a chord. Make a conjecture about the number
of regions in the interior of the circle made by the chords and test your conjecture.

For n = 1; 2; 3; 4; 5, we draw actual circles and count the number of regions made by the chords obtained
by connecting every pair of points.

S
DM
We summarize the number of regions in the following table.

n
no. of regions
1
1
2
2
3
4
4
8
5
16

If we observe the pattern on the number of regions, they seem to be powers of 2. In fact, for
n = 1; 2; 3; 4; 5, the number of regions in the circle is 2n−1 . It is therefore reasonable for use to
give the following conjecture.
P
Conjecture. The number of regions in the interior of the circle made by connecting every pair of points
in a set of n points in the circumference is 2n−1 .
PU

The best way to test the conjecture is to check the example for the next larger n, which is n = 6.
Constructing the circle for n = 6 and counting the regions,

we see that the number of regions is 31 and not 26−1 . This counterexample disproves our conjecture.

All Rights Reserved. 2020 Abdul, Atienza, et. al.


Lesson 3 36

Definition 13
Deductive reasoning is the process of reasoning that arrives at a conclusion based on previously
accepted general statements.

Deductive reasoning does not rely on examples. We make our conclusion based on general statements
whose truth value is known or assumed. Formal mathematics is usually based on this type of reasoning.
We first lay down definition of terms, and assume basic true statements called axioms and derive true
statements from these axioms called as theorems.

Example 23. The following are examples of deductive reasoning.

S
1. Starfish are invertebrates. Patrick is a starfish. Therefore, Patrick is invertebrate.
√ √
2. Every rational number is a real number. The number −1 is not real. Therefore, −1 is not
rational.

DM
Inductive reasoning cannot in general prove general statements as this relies on examples only. In contrast,
we can use deductive reasoning to prove a certain conjecture.

Example 24. Choose any number. Multiply by 3. Add 6 to the result. Divide the result by 3. Finally,
subtract the original number from the result of the previous step. Use inductive reasoning to make a
conjecture about the final result and use deductive reasoning to prove the conjecture.

Solution. We first consider few examples.


P
test number 9 15 28
multiply by 3: 27 45 84
PU

add 6: 33 51 90
divide by 3: 11 17 30
subtract the orig. no. 2 2 2

We see that based from the three test numbers, the final results are the same and are all equal to 2.
There is a reason to conjecture that the final result will always be 2 regardless on where we start. To
prove this claim, take an arbitrary number x.
multiply by 3: 3x
add 6: 3x + 6
3x + 6
divide by 3: =x +2
3
subtract the orig. no: (x + 2) − x = 2.
Therefore, as claimed, it is now proven that we will always end up with 2.

All Rights Reserved. 2020 Abdul, Atienza, et. al.


Lesson 3 37

3.2 George Polya’s Guidelines for Problem Solving


In 1945, mathematician George Pólya devised a model for problem solving and published it in his book
How to Solve It. The book contains a collection of mathematical problems and selected strategies on
dealing these. His problem solving model, which he called heuristic (or serving to discover), is as follows.

POLYA’S FOUR STEPS:

1. Understand the problem. Ask questions, experiment, or otherwise rephrase the question in
your own words.

S
2. Devise a plan. Find the connection between the data and the unknown. Look for patterns, relate
to a previously solved problem or a known formula, or simplify the given information to give you
an easier problem.

1. Draw a diagram.

2. Solve a simpler problem.


DM
3. Carry out the plan. Check the steps as you go.

4. Look back. Examine the solution obtained. In other words, check your answer.

Together with these guidelines, the following are some of his recommended strategies:

5. Guess and check.

6. Find a pattern.

3. Make a table. 7. Use a formula or an equation.


P
4. Work backwards. 8. Using logical reasoning.

Example 25. In a seminar, 30 attendees were present. During their meet-and-greet activity, they were
PU

asked to have a handshake with everyone in the room. If each one did handshake with everyone, how
may handshakes took place?

Solution.

Step 1. Understand the Problem. There were 30 attendees present. A simple handshake means
letting a distinct pair be recognized. Moreover, if A shakes hands with B, then B shakes hands
with A as well.

Step 2. Devise a plan. We start with solving simpler cases, say 3, 4 and 5 persons. We can draw
a diagram where a person is represented by nodes while handshakes by arcs connecting the
nodes. From here, we try to find a pattern.

Step 3. Carry out the plan. The following figures represent the handshakes that took place among
3, 4 and 5 persons.

All Rights Reserved. 2020 Abdul, Atienza, et. al.


Lesson 3 38

A group of 3 persons makes 3 handshakes, a group of 4 persons makes 6 handsakes and a


group of 5 persons makes 10 handshakes. Now, for each case with k persons, each of these
persons has to have a handshake with the other k − 1 persons. So, the product k(k − 1) is

S
the number of all handshakes from individual perspective. Note that if A shakes hands with B,
then B shakes hands with A as well. Thus, only half of k(k − 1) represents the total number of
k(k − 1)
handshakes. Hence, a pattern is generalized by will lead to the number of handshakes

DM
2
=
2
2
that took place in a group of k persons. Therefore, there were a total of

k(k − 1) 30(30 − 1)
= 435 handshakes.

Step 4. Look back. Every person will be shaking hands with 29 other. Thus, 870 handshakes are
noted for individual perperspective. Half of which is 435.
P
Example 26. Andrew has some magic cards to trade. Ian has 2 more than 2 times the number of magic
cards Andrew has. Patrick has 2 less than Ian. Ken has 4 less than 2 times the number of magic cards
PU

Patrick has. Patrick has 8 magic cards. How many magic cards does Andrew have to trade?

Solution.

Step 1. Understand the Problem. The number of magic cards Ian has depends on the number of
magic cards Andrew has. The number of magic cards Patrick has depends on the number of
magic cards Ian has. The number of magic cards Ken has depends on the number of magic
cards Patrick has. It is clear that 8 cards are in Patrick’s possession.

Step 2. Devise a plan. We can settle this by working backwards starting from the number of magic
cards Patrick has. Making a table may aid organization.

Step 3. Carry out the plan.

All Rights Reserved. 2020 Abdul, Atienza, et. al.


Lesson 3 39

Statements of Hints Arithmetic Sense Remarks


Patrick has 8 magic cards. 8 This is the last event.
Ken has 4 less than 2 times Operation is not
the number of magic cards (2 × 8) − 4 = 12 yet revered.
Patrick has. Ken has 12 cards.
Patrick has 2 less Operation is reversed.
less than Ian. 8 + 2 = 10 Ian has 10 cards.
Ian has 2 more than Operation is reversed.
2 times the number of (10 − 2)=2 = 4 Andrew has 4 cards.
magic cards Andrew has.

S
Step 4. Look back. If Andrew has 4 magic cards, then Ian has 2 more than twice of 4 or 10 magic
cards. If Ian has 10 magic cards, then Patrick has 10 - 2 = 8 magic cards. Lastly, if Ken has

DM
4 less than twice of 8 of 12 magic cards.
P
PU

All Rights Reserved. 2020 Abdul, Atienza, et. al.


Lesson 3 40

Assessment
1. Explain why you can never be sure that a conclusion you arrived at using inductive reasoning is
true.

2. Select any two-digit number. Multiply it by 9. Then add the digits. Keep adding the digits in
the answer until you get a single-digit answer. Using inductive reasoning, what can you conjecture
about any whole number multiplied by 9? Use deductive reasoning to prove that your conjecture
is true.

3. Use Polya’s Four Steps to solve the following problems.

S
(a) Susie’s age this year is a multiple of 5. Next year, her age is a multiple of 7. What is her
present age?

DM
(b) Consider a square whose side is 1 unit. If the measure of its side is doubled, what will be its
new area as compare to the smaller square? How about if the side of the smaller square was
tripled, what will be its new area?
(c) How many perfect squares are there between 1,000,000 and 9,000,000?
(d) Determine the number of different triangles that can be drawn given eight noncollinear points?
(e) There are 25 students asked by their literature instructor regarding with the type of literary
works they prefer to read. He found out that 10 prefer to read novels, 11 prefer to read short
stories, 15 prefer to read poems, 5 for both novels and short stories, 4 both short stories and
P
poems, 7 for both novels and poems, and 3 prefer all. How many students prefer none of the
given types of literary works?
PU

All Rights Reserved. 2020 Abdul, Atienza, et. al.


Lesson 4 41

Lesson 4: Statistics and Data Management

Learning Outcomes
At the end of the lesson, the students are able to

1. demonstrate the ability to apply fundamental concepts in exploratory data analysis;

2. define the field of Statistics in terms of its definition and application;

3. enumerate the procedures involved in collecting data;

S
4. distinguish between the nominal, ordinal, interval and ratio methods of data measurement;

5. recognize the various ways to present data;

DM
6. identify the features that describe a data distribution.

Statistics is the study of the collection, organization, analysis, interpretation, and presentation of data.
It deals with all aspects of data, including the planning of its collection in terms of the design of
surveys and experiments. Some consider statistics a mathematical body of science that pertains to the
collection, analysis, interpretation or explanation, and presentation of data, while others consider it a
branch of mathematics concerned with collecting and interpreting data. Because of its empirical roots
and its focus on applications, statistics is usually considered a distinct mathematical science rather than
a branch of mathematics.
P
4.1 Basic Concepts
PU

Statistics is defined as a branch of mathematics which is concerned with facilitating wise decision-
making in the face of uncertainty and that, therefore develops and utilizes techniques for collection,
effective presentation, and proper analysis of data.

Branches of Statistics

1. Descriptive Statistics is concerned with the description and summarization of data, It deals with
the techniques used in the collection, presentation, organization, and analysis of the data on hand.

2. Inferential Statistics is concerned with the drawing of conclusions from data. It deals with the
techniques used in generalizing from samples to populations, performing estimations and hypothesis
tests determining relationships among variables, and making predictions.

All Rights Reserved. 2020 Abdul, Atienza, et. al.


Lesson 4 42

Functions of Statistics

1. Condensation. Generally speaking by the verb ‘to condense’, we mean to reduce or to lessen.
Condensation is mainly applied at embracing the understanding of a huge mass of data by providing
only few observations.

2. Comparison. Classification and tabulation are the two methods that are used to condense the
data. They help us to compare data collected from different sources. Grand totals, measures
of central tendency measures of dispersion, graphs and diagrams, coefficient of correlation, etc.
provide ample scope for comparison. As statistics is an aggregate of facts and figures, comparison

S
is always possible and in fact comparison helps us to understand the data in a better way.

3. Forecasting. By the word forecasting, we mean to predict or to estimate beforehand. Given the

DM
data of the last ten years connected to the number of students enrolled in PUP, it is possible to
predict or forecast the number of students that will enroll for the near future. In business also
forecasting plays a dominant role in connection with production, sales, profits etc. The analysis of
time series and regression analysis plays an important role in forecasting.

4. Estimation. One of the main objectives of statistics is drawn inference about a population from
the analysis for the sample drawn from that population.

5. Tests of Hypothesis. A statistical hypothesis is some statement about the probability distri-
bution, characterizing a population on the basis of the information available from the sample
P
observations. In the formulation and testing of hypothesis, statistical methods are extremely use-
ful. Whether the grades of students increased because they are motivated or whether the new
teaching method is effective in discussing a particular topic are some examples of statements of
hypothesis and these are tested by proper statistical tools.
PU

Scope of Statistics

1. Statistics and Industry. Statistics is widely used in many industries. In industries, control charts
are widely used to maintain a certain quality level. In production engineering, to find whether the
product is conforming to specifications or not, statistical tools, namely inspection plans, control
charts, etc., are of extreme importance. In inspection plans we have to resort to some kind of
sampling - a very important aspect of Statistics.

2. Statistics and Commerce. Statistics are lifeblood of successful commerce. Any businessman
cannot afford to either by under stocking or having overstock of his goods. In the beginning he
estimates the demand for his goods and then takes steps to adjust with his output or purchases.
Thus statistics is indispensable in business and commerce.

All Rights Reserved. 2020 Abdul, Atienza, et. al.


Lesson 4 43

3. Statistics and Economics. Statistical methods are useful in measuring numerical changes in
complex groups and interpreting collective phenomenon. Nowadays the uses of statistics are abun-
dantly made in any economic study. Both in economic theory and practice, statistical methods
play an important role.

4. Statistics and Education. Statistics is widely used in education. Research has become a
common feature in all branches of activities. Statistics is necessary for the formulation of policies
to start new course, consideration of facilities available for new courses etc. There are many people
engaged in research work to test the past knowledge and evolve new knowledge. These are possible
only through statistics.

S
5. Statistics and Planning. Statistics is indispensable in planning. In the modern world, which can
be termed as the “world of planning”, almost all the organizations in the government are seeking

DM
the help of planning for efficient working, for the formulation of policy decisions and execution of
the same. In order to achieve the above goals, the statistical data relating to production, consump-
tion, demand, supply, prices, investments, income expenditure etc and various advanced statistical
techniques for processing, analyzing and interpreting such complex data are of importance. In
India statistics play an important role in planning, commissioning both at the central and state
government levels.

6. Statistics and Medicine. In Medical sciences, statistical tools are widely used. In order to test
the efficiency of a new drug or medicine, t - test is used or to compare the efficiency of two drugs
or two medicines, t-test for the two samples is used. More and more applications of statistics are
P
at present used in clinical investigation.

7. Statistics and Modern Applications. Recent developments in the fields of computer technol-
ogy and information technology have enabled statistics to integrate their models and thus make
PU

statistics a part of decision making procedures of many organizations. There are so many software
packages available for solving design of experiments, forecasting simulation problems etc.

Limitations of Statistics

1. Statistics is not suitable to the study of qualitative phenomenon. Since statistics is


basically a science and deals with a set of numerical data, it is applicable to the study of only
these subjects of enquiry, which can be expressed in terms of quantitative measurements. As a
matter of fact, qualitative phenomenon like honesty, poverty, beauty, intelligence etc, cannot be
expressed numerically and any statistical analysis cannot be directly applied on these qualitative
phenomenon.

2. Statistics does not study individuals. Statistics does not give any specific importance to the
individual items; in fact it deals with an aggregate of objects. Individual items, when they are taken

All Rights Reserved. 2020 Abdul, Atienza, et. al.


Lesson 4 44

individually do not constitute any statistical data and do not serve any purpose for any statistical
enquiry.

3. Statistical laws are not exact. It is well known that mathematical and physical sciences are
exact. But statistical laws are not exact and statistical laws are only approximations. Statistical
conclusions are not universally true. They are true only on an average.

4. Statistics table may be misused. Statistics must be used only by experts; otherwise, statistical
methods are the most dangerous tools on the hands of the inexpert. The use of statistical tools
by the inexperienced and untraced persons might lead to wrong conclusions.

S
5. Statistics is only one of the methods of studying a problem. Statistical method do
not provide complete solution of the problems because problems are to be studied taking the

Population and Sample DM


background of the countries culture, philosophy or religion into consideration. Thus the statistical
study should be supplemented by other evidences.

In statistics, we are often interested in gathering information from a group of objects. If the group
in consideration consists of large number of objects, we try to obtain information about the group by
examining its subgroup.
P
Definition 14
The total collection of all the elements that we are interested in is called a population. A
subgroup of the population that will be studied in detail is called a sample.
PU

In order for the data from the sample is informative about the population, it must be representative
of the population. Being representative of the population does not mean that the characteristic of the
sample is exactly that of the total population, but instead the sample was obtain in such way that every
member of the population had an equal chance to be included in the sample.

Definition 15
A sample of k members of a population is called a random sample, also called a simple random
sample, if the members are chosen in such a way that all possible choices of the k members are
equally likely.

After a random sample is obtain from the population, we can use statistical inference to draw general-
izations about the population by examining the members of the sample.

All Rights Reserved. 2020 Abdul, Atienza, et. al.


Lesson 4 45

4.2 Steps in Statistical Investigation


1. Defining the problem

(a) Identify a specific problem.


(b) Define the scope and limitations, assumptions to be made, and expected outcomes.

2. Collection of data

(a) Make sure to collect the data properly.


(b) Incomplete, fabricated, outdated, and inaccurate data are useless.

S
3. Summarization and tabulation of data

(a) This refers to organization of data in text, tables, graphs and charts, so that logical conclusion
can be derived from them.

4. Analysis of data
DM
(b) Explore the data to obtain additional insight that could contribute to the study.

(a) This pertains to the process of deriving from the given data relevant information from which
numerical descriptions can be formulated.
(b) Summarized data must be examined so that insights and meaningful information ca be pro-
duced to support decision-making or solutions to the question or problem at hand.
P
5. Interpretation of data and results

(a) Refers to the task of drawing conclusions from the analyzed data.
(b) Results must be able to answer the research problem and give recommendations.
PU

6. Presentation of the result

(a) Present all pertinent results in a clear and concise manner.


(b) Use appropriate form of media to present results.

4.3 Sampling and Sampling Techniques


Sampling refers to the process of obtaining samples from the population. Sampling maybe categorized as
either probability sampling or non-probability sampling. Probability sampling, also referred to as random
sampling, is the method of sampling in which every member of the population have equal chance of
being selected as sample; otherwise, it is considered as non-probability sampling. We should note that in
able to properly use the techniques of statistical inference, probability sampling must be used to obtain
samples.

All Rights Reserved. 2020 Abdul, Atienza, et. al.


Lesson 4 46

Probability Sampling Techniques

1. Simple Random Sampling. A probability sampling technique wherein all possible subsets con-
sisting of n elements selected from the N elements of the population have the same chances of
selection.

2. Systematic Sampling. This is a probability sampling technique wherein the selection of the
first element is at random and the selection of other elements in the sample is systematic by
subsequently taking every kth element from the random start where k is the sampling interval.

3. Stratified Random Sampling. A probability sampling method where we partition the population

S
into non-overlapping strata or group and then a proportional sample is chosen from each strata.
The actual sample is the sum of the samples derived from each strata.

DM
4. Cluster Sampling. A probability sampling technique wherein we partition the population into
non-overlapping groups or clusters consisting of one or more elements, and then select a sample
of clusters. Every member of the selected cluster will be considered as sample.

Non-Probability Sampling Techniques

1. Accidental Sampling. Sample is chosen by the researcher by the obtaining members of the
population in a convenient, often haphazard way.

2. Quota Sampling. There is specified number of persons of certain types is included in the sample.
The researcher is aware of categories within the population and draws samples from each category.
P
The size of each categorical sample is proportional to the proportion of the population that belongs
in that category.
PU

3. Purposive Sampling. The researcher employs his or her judgments on choosing which he or she
believes are representative of the population.

4. Snowball Sampling. This technique is also called referral sampling. A primary set of samples
are chosen based on the criteria set by the researcher. Information on where to find succeeding
set of sample having the same criteria will be gathered from this primary set in order to expand
the number of samples.

4.4 Sample Size Considerations


The sample size is typically denoted by n and it is always a positive integer. No exact sample size can be
mentioned here and it can vary in different research settings. However, all else being equal, large sized
sample leads to increased precision in estimates of various properties of the population.
To determine the sample size we can apply one of the following methods:

All Rights Reserved. 2020 Abdul, Atienza, et. al.


Lesson 4 47

1. Slovin’s Formula. Slovin’s formula is used to calculate the sample size n given the population
size and a margin of error E. It is a formula use to estimate sampling size of a random sample
from a given population. We can compute

N
n= ;
1 + NE 2

where N is the population size.

Example 27. A researcher plans to conduct a survey about food preference of BS Stat students. If the
population of students is 1000, use the Slovin’s formula to find the sample size if the margin of error is 5%.

S
Solution. Using the Slovin’s formula, we get

DM
n=
1000
1 + 1000(0:05)2
≈ 285:71:

Therefore, the researcher needs to survey 286 BS Stat Students.

2. Minimum Sample Size for Estimating a Population Mean. The estimated minimum sample
size n needed to estimate a population mean — to within E units at 100(1 − ¸)% confidence is

(z¸=2 )2 ff 2
n= ;
E2
where ff is the known population standard deviation, E is the margin of error and z¸=2 is a value
P
which can be obtained in the z-table.

Example 28. Suppose we want to know the average age of STEM students. We would like to be 99%
PU

confident about our results. From previous study, we know that the standard deviation for the population
is 1.3. How many students should be chosen for a survey if the margin of error is 0.2.

Solution. Find z¸=2 by looking at the z-table.

¸ = (1 − 0:99) = 0:01 =⇒ z¸=2 = z0:005 :

The closest z-score for 0:005 in the z-table is 2:58. Thus,

(2:58)2 (1:3)2
n= ≈ 281:23:
(0:2)2

which we round up to 282, since it is impossible to take a fractional observation. We need a 282 STEM
students as a sample for our study.

All Rights Reserved. 2020 Abdul, Atienza, et. al.


Lesson 4 48

3. Minimum Sample Size for Estimating a Population Proportion The estimated minimum
sample size n needed to estimate a population proportion p to within E at 100(1 − ¸)% confidence
is
(z¸=2 )2 p̂(1 − p̂)
n= :
E2
This is also called the Cochran Formula.

The dilemma here is that the formula for estimating how large a sample to take contains the
number p̂, which we know only after we have taken the sample. There are two ways out of this
dilemma.

S
• First, typically the researcher will have some idea as to the value of the population proportion

in the formula.
DM
p, hence of what the sample proportion p̂ is likely to be. For example, if last month 37% of
all voters thought that state taxes are too high, then it is likely that the proportion with that
opinion this month will not be dramatically different, and we would use the value 0.37 for p̂

• The second approach to resolving the dilemma is simply to replace p̂ in the formula by 0.5.
This is because if p̂ is large then 1 − p̂ is small, and vice versa, which limits their product to
a maximum value of 0.25, which occurs when p̂ = 0:5. This is called the most conservative
estimate, since it gives the largest possible estimate of n.
P
Example 29. Suppose we are doing a study on the inhabitants of a large town, and want to find out
how many households serve breakfast in the mornings. We don’t have much information on the subject
to begin with, so we’re going to assume that half of the families serve breakfast: this gives us maximum
PU

variability. Here, p̂ = 0:5. We want 95% confidence and at least 5% precision.

Solution. Find z¸=2 in the z-table. We have

¸ = (1 − 0:95) =⇒ z¸=2 = z0:025 :

The closest z-score for 0:025 in the z-table is 1:96. A 95% confidence level gives us Z values of 1.96,
we get
(1:96)2 (0:5)(1 − 0:5)
n= ≈ 384:16:
(0:05)2
Hence, a random sample of 385 households in our target population should enough to give us the
confidence levels we need.

All Rights Reserved. 2020 Abdul, Atienza, et. al.


Lesson 4 49

Finite Population Correction for Proportions

If the population is small then the sample size can be reduced slightly. This is because a given sample size
provides proportionately more information for a small population than a large population. The formula
is
n0
n= ;
n0 − 1
1+
N
where n0 is the Cochran’s sample size recommendation, N is the population size and n is the new adjusted
sample size.

S
Example 30. In the preceding example, if there were just 1000 households in the target population, we
would calculate
385
n= ≈ 278:18:
385 − 1

DM
1+
1000
All we need are 279 households in our sample, a substantially smaller sample size.

4.5 Methods of Data Collection


1. Survey Method. The survey is a method of collecting data on the variable of interest by asking
people questions. This may be done, by interview or by using questionnaires.

2. Observation. Observation is a method of obtaining data or information by using our primary


senses.
P
3. Experiment. Experiment is a method of collecting data where there is direct human intervention
on the conditions that may affect the values of the variable of interest.
PU

4.6 Levels of Measurement


1. The nominal level of measurement classifies data into mutually exclusive (non-overlapping)
categories in which no order or ranking can be imposed on the data.

Example: Gender (male, female), Zip Code, Color, Nationality, Political affiliation, Religious
affiliation.

2. The ordinal level of measurement classifies data into categories that can be ranked; however,
precise differences between the ranks do not exist.

Example: Grade(A,B,C,D,F), Rating Scale/Likert scale, Ranking of tennis players, Judging (First
place, second place, etc.

All Rights Reserved. 2020 Abdul, Atienza, et. al.


Lesson 4 50

3. The interval level of measurement ranks data, and precise differences between units of measure
do exist; however, there is no meaningful zero.

Example: Temperature, IQ, SAT score

4. The ratio level of measurement possesses all the characteristics of interval measurement, and
there exists a true zero. In addition, true ratios exist when the same variable is measured on two
different members of the population

Example: Height, Weight, volume, Time, Salary, Age

S
4.7 Presentation of Data

DM
After data have been collected, the researcher can now present them in the following logical methods.

1. Textual Form. Data are presented in paragraph of text. The text highlights the important figures
or results that the researcher wishes to focus on.

2. Tabular Form. Data appears in a systematic manner in rows and columns.


The following is an example of a Simple or One-Way Table.

Table 1
Frequency Distribution of the
P
Students Enrolled for the Last 6 Years
Year Frequency
2012 13,450
PU

2013 13,200
2014 15,389
2015 16,790
2016 18,900
2017 19,500
Total 97,229

All Rights Reserved. 2020 Abdul, Atienza, et. al.


Lesson 4 51

The following is an example of a Two-Way Table.

Table 2
Number of Students Enrolled for the Last 6 Years
When Grouped According to Sex

Year
Sex
2012 2013 2014 2015 2016 2017 Total
Male 5560 6095 7386 8056 7945 6451 41493
Female 7890 7105 8003 8734 10955 13049 55736

S
Total 13450 13200 15389 16790 18900 19500 97229

3. Graphical Form. Data or relationship among variables could be presented in visual form, thru

Types of Statistical Charts DM


graph or diagrams. In that manner, the reader can easily perceive what is being meant by the
figure or any trend being portrayed by the data.

(a) Bar Graph (Vertical Bar/Column Charts) is applicable for showing comparison of
amount of a variable of interest collected over time.

Simple Chart
P
PU

Grouped Column Charts

All Rights Reserved. 2020 Abdul, Atienza, et. al.


Lesson 4 52

Subdivided Column Charts

S
(b) Histogram is similar to the bar graph but the base of the rectangle has a length exactly
equal to the class width of the corresponding interval. Also, there are no spaces between
rectangles.

DM Histogram
P
(c) Pictograph is similar to the bar chart but instead of bars, we use pictures or symbols to
represent a value or an amount.
PU

Pictograph

(d) Pie Chart is a circular graph partitioned into several section, depicting relative percentage
with respect to the total distribution.

All Rights Reserved. 2020 Abdul, Atienza, et. al.


Lesson 4 53

Pie Chart

S
(e) Line Graph is a graph used to visualize data that changes continuously over time.

Simple Line Graph

DM Multiple Line Graph


P
PU

(f) Statistical Map is used to show data in geographical areas.

Statistical Map

All Rights Reserved. 2020 Abdul, Atienza, et. al.


Lesson 4 54

4.8 Measures of Central Tendency


A measure of central tendency or average is a location measure that pinpoints the center or typical
middle value of a data set. A convenient way of describing a set of data with a value that describes
the average characteristic a data set. The three common measures of central tendency are the mean,
median and mode.

Mean

Definition 16

S
Suppose that a variable x assumes values x1 ; x2 ; : : : ; xn . The arithmetic mean x of these values
is defined as n
1X x1 + x2 + · · · + xn
P
x
x=

DM
= xi = :
n n i=1 n

The (arithmetic) mean of x is obtained by adding all its observed values and dividing the sum by the
total number of observations.

Example 31. The scores of 15 students in Mathematics in the Modern World on an exam consisting
of 25 items are 25,20,18,18,17,15,15,15,14,14,13,12,12,10,10. Determine the mean score for this exam.

Solution. Let x denote the score of a random student from the sample of 15 students in Mathematics in
the Modern World. The sum of these scores is x = 228. Hence, the mean score of the 15 students is
P
P
228
P
x
x= = = 15:2:
n 15
PU

There are cases when the observations in a data set assume respective weights. In this case where the
weights are positive integers, we can call these weights as frequencies. The following gives a formula
for the weighted mean of a weighted data set.

Definition 17
Given the x values x1 ; x2 ; : : : ; xn assuming respective weights w1 ; w2 ; : : : ; wn , the weighted mean
is defined as
w1 x1 + w2 x2 + · · · + wn xn
P
wx
x= P = :
x w1 + w2 + · · · + wn

Example 32. Suppose that we are asked to get the mean of the data set 1; 1; 3; 3; 3; 3; 4; 4; 4; 6; 6; 8.

All Rights Reserved. 2020 Abdul, Atienza, et. al.


Lesson 4 55

Using the original formula for the arithmetic mean we find that

(1 + 1) + (3 + 3 + 3 + 3) + (4 + 4 + 4) + (6 + 6) + 8
x=
12
2·1+4·3+3·4+2·6+1·8
=
1+4+3+2+1
2 + 12 + 12 + 12 + 8
=
12
46
=
12
= 3:833

S
We can interpret the mean of the data values as the fulcrum or center of gravity in a balance scale as
shown below.

1
DM
P
1 2 3 4 5 6 7 8

mean = 3:8333
PU

Example 33.
Calculate the General Weighted Average (GWA) of
Course Grade Units
Julius Garde for the first semester of school year
BM 112 1.25 3
2019-2020 as shown in the following table.
BM 101 1.00 3
AC 103 1.25 6
Solution. To solve for the GWA, we first consider
MG 101 1.00 3
the entries on the second column of the table as the
EC 111 1.50 3
points xi and the entries in the third column as the
MK 101 1.50 3
corresponding weights wi . By constructing a fourth
FM 111 1.20 3
column consisting of the products wi xi and finding
PE 1 1.00 2
the column totals, we get the table below.

All Rights Reserved. 2020 Abdul, Atienza, et. al.


Lesson 4 56

Course xi wi wi xi
BM 112 1.25 3 3.75
BM 101 1.00 3 3.00
AC 103 1.25 6 7.50
MG 101 1.00 3 3.00
EC 111 1.50 3 4.50
MK 101 1.50 3 4.50
FM 111 1.20 3 3.60
PE 1 1.00 2 2.00

S
Total w = 26 w x = 32:00
P P

We see from the column totals that w = 26 and w x = 32. Therefore, the weighted mean or the
P P

DM
general weighted average (GWA) of Julius Garde for the first semester of AY 2019-2020 is

32
P
wx
x= P = = 1:23:
w 26

Median

Definition 18
The median, usually denoted by x̃, is the middle value of a data set if the observations are
P
arranged either in increasing or decreasing order.

Outliers in the data set do not affect the median. Thus, the median is preferred over the mean as a
PU

measure of central tendency when the data contains outliers. To find the median, begin by listing the
data in order from smallest to largest, or largest to smallest.

If the number of data values, N, is odd, then the median is the middle data value. This value can be
found by rounding N=2 up to the next whole number. If the number of data values is even, there is no
one middle value, so we find the mean of the two middle values (values N=2 and N=2 + 1)

Example 34. Given the scores of 15 students in Mathematics in the Modern World on an exam consisting
of 25 items:
25; 20; 18; 18; 17; 15; 15; 15; 14; 14; 13; 12; 12; 10; 10

Since the data is already arranged in decreasing order and there are 15 observations, hence, we round
15
up = 7:5 to the nearest whole number, which is 8, and take the 8th observation from the left (or
2
right). Therefore, the median is x̃ = 15: In comparison to example 31, the computed mean is 15:2.

All Rights Reserved. 2020 Abdul, Atienza, et. al.


Lesson 4 57

10 11 12 13 14 15 16 17 18 19 20 21 22 23 24 25 26

S
mean

median

Month
January
February
March
April
Hours Lost
55
23
24
37
DM
Remark. In general, the median need not equal the mean.

Example 35. The data given below is the total number of hours lost due to tardiness and absences of
employees in a company in a given year. Find the median.

Solution. If the data are arranged in increasing order, we have

20; 23; 24; 27; 30; 32; 37; 37; 40; 48; 42; 55:

May 37
June 48
Since there are 12 observations (even), we take note of the two
P
July 42 middle observations then compute
August 27
September 20
32 + 37
October 40 x̃ = = 34:5:
November 30 2
PU

December 32

Therefore, the median number of hours lost due to tardiness and absences of employees in a company
in the given year is 34:5 hours.

Mode

Definition 19
The mode is the most frequent observation in a given data set.

Outliers in the data set do not affect the mode. It is possible that the mode of a data set does not
exist, and it is not always unique. It is an appropriate measure of average for data measured only in the
nominal level. We will denote mode using the symbol x̂.

All Rights Reserved. 2020 Abdul, Atienza, et. al.


Lesson 4 58

Example 36. Suppose that we wanted to know the “average color” of cars used by the residents in a
given village. In our vehicle color survey, we collected the following data.
Color Frequency
Blue 3
Green 5
Red 4
White 3
Black 2
Grey 3

S
Since color of vehicles are measured up to the nominal level, the most appropriate measure for the
“average color” is then the mode. The most frequent color is Green, a total of 5 vehicles. Therefore, the
“average color” in our survey data must be Green.

4.9
DM
It is possible for a given data set to have more than one modes. Such a data set is said to be multimodal.
If a given set has only one mode, the data set is unimodal. If it has two modes, the data set is bimodal,
and so on.

Measures of Dispersion or Variability


Measures of dispersion are descriptive summary measures that helps us characterize the data set in terms
of how varied the observations are from the center. If its value is small, then this indicates that the
observations are not too different from the center. On the other hand, if its value is large, then this
indicates that the observations are very different from the center or that they are widely spread out from
P
the center.

Range
PU

Definition 20
The range is the difference between the largest and the smallest observations or items in a set of
data.

The range of a data set is easy to compute, but it is a limited measure because it depends on only two
of the numbers (the highest and the lowest) in the data set. Hence, the range can easily be affected
by outliers. Also, it does not provide any information regarding the concentration of the data from the
center.

Example 37. The following are scores of 20 coming from two different sections, 10 from each section,
in a 50-item exam in MMW.
section 1 40 38 42 40 39 39 43 40 39 40
section 2 46 37 40 33 42 36 40 47 34 45

All Rights Reserved. 2020 Abdul, Atienza, et. al.


Lesson 4 59

For section 1, the highest score is 43, while the lowest score is 38. Thus,

range = 43 − 38 = 5:

On the other hand, for section 2, the highest score is 47, while the lowest score is 33. Thus,

range = 47 − 33 = 14:

Therefore, the scores of students surveyed from section 2 gets a wider range than those of students
surveyed from section 1.

S
Variance and Standard Deviation

Suppose that the center of a population data set {x1 ; x2 ; : : : ; xN } is best described by the arithmetic

would like to compute for

i=1
DM
mean — and that our goal is to get the average “distance” of each data point xi form —. Naturally, we

1 X

(xi − —) =
N

N i=1
(xi − —):

However, using the properties of summations, and the fact that n— = x1 + x2 + · · · + xN we can check
that
N N
X

i=1
xi −
N
X

i=1
— = N— − N— = 0:

In other words, the sum of the deviations from the mean is 0, and therefore, we cannot have a meaningful
measure of variability this way. The reason behind this fact is that some of the deviations from the mean
P
are negative (those which are to the left of the mean) and some are positive (those which are to the right
of the mean) and they cancel each other out. However, we can work our way out of this unfortunate
situation if we can ignore the signs of these deviations. One way to do this is to take the square these
PU

deviations from the mean. We then have the following definition.

Definition 21
The variance of a population data set {x1 ; x2 ; : : : ; xN } with population mean — is defined as

N
1 X
ff 2 = (xi − —)2 :
N i=1

On the other hand, the variance of a sample data set {x1 ; x2 ; : : : ; xn } with sample mean x is
defined as n
2 1 X
s = (xi − —)2 :
n − 1 i=1

As we may have noticed, the formula for the sample variance differs significantly from the formula for

All Rights Reserved. 2020 Abdul, Atienza, et. al.


Lesson 4 60

the population variance mainly because of the divisor n − 1. The reason behind this is rather technical
and mathematical in nature. Simply taken, the divisor n − 1 removes the “bias” in s 2 when we want it
to estimate ff 2 for the purposes of making inferences.

Notice that the variance is a nonnegative quantity because it came from averaging squared quantities.
We also realize that there is one major drawback to using the variance. If we follow the steps in calcu-
lating the variance, we find that the variance is measured in terms of square units because we took the
squares of the deviation. For example, if our sample data is measured in terms of meters, then the units
for a variance would be given in square units.

S
In order to standardize the units, we can take the square root of the variance to eliminate the problem of

Definition 22
DM
squared units, and gives us a measure of the spread that will have the same units as our original sample
or population data.

The population (sample) standard deviation is the nonnegative square root of the the pop-
ulation (sample) variance. In symbols,
√ √
ff = ff 2 and s = s 2:
P
PU

Example 38. Using the sample data sets in example 37, determine which section exhibits a greater
variability in terms of standard deviations.

Solution. Let x denote the scores of students sampled from section 1 and let y denote the scores of
students sampled from section 2. To calculate the standard deviations of each sample, we first take note
that the sample means from each section are

400 400
P P
x y
x= = = 40 and y = = = 40:
n 10 n 10

To calculate the sample standard deviation, we construct the following table.

All Rights Reserved. 2020 Abdul, Atienza, et. al.


Lesson 4 61

x y x −x y −y (x − x)2 (y − y )2
40 46 0 6 0 36
38 37 −2 −3 4 9
42 40 2 0 4 0
40 33 0 −7 0 49
39 42 −1 2 1 4
39 36 −1 −4 1 16
43 40 3 0 9 0
40 47 0 7 0 49
39 34 1 36

S
−1 −6
40 45 0 5 0 25
x = 400 y = 400 (x − x)2 = 20 (y − y )2 = 224
P P P P

s =

DM
Therefore, the sample variance for the sample from section 1 is

2
P
(x − x)2
n−1

while the sample variance for the sample from section 2 is

2
s =
P
(y − y )2
n−1
=
=
20

9
9

224
= 2:2222;

= 24:8888:

Taking square roots, we find that the sample standard deviations of section 1 and section 2 respectively
√ √
are 2:2222 ≈ 1:49 and 24:8888 ≈ 4:99. We can conclude that for these samples, the one from
P
section 1 exhibits the lesser variability than that from section 2. We comment that even though the two
samples have equal means, the standard deviations showed the actual difference between the two data
sets.
PU

All Rights Reserved. 2020 Abdul, Atienza, et. al.


Lesson 4 62

Assessment
1. A research objective is presented. For each,identify the (a)population and (b) sample in the study.

(a) A polling organization contacts 2141 male university graduates who have a white-collar job
and asks whether or not they had received a raise at work during the past 4 months.
(b) A quality-control manager randomly selects 70 bottles of ketchup that were filled on July 17
to assess the calibration of the filling machine.
(c) Every year the PSA releases the Current Population Report based on a survey of 50,000
households. The goal of this report is to learn the demographic characteristics, such as

S
income, of all households within the Philippines.

2. Determine the level of measurement of each variable.

DM
(a) birth order among siblings in a family
(b) favorite movie
(c) volume consumption of water used by a household in a day
(d) eye color
(e) number of siblings

3. Determine the type of sampling used.

(a) A member of Congress wishes to determine her constituents’ opinion regarding estate taxes.
P
She divides her constituency into three income classes: low-income households, middle-income
households, and upper-income households. She then takes a simple random sample of house-
holds from each income class.
PU

(b) A college official divides the student population into five classes: freshman, sophomore, junior,
senior, and graduate student. The official takes a simple random sample from each class and
asks the members opinions regarding student services.
(c) The presider of a guest-lecture series at a university stands outside the auditorium before a
lecture begins and hands every fifth person who arrives, beginning with the third, a speaker
evaluation survey to be completed and returned at the end of the program.
(d) To determine his DSL Internet connection speed, Shawn divides up the day into four parts:
morning, midday, evening, and late night. He then measures his Internet connection speed
at 5 randomly selected times during each part of the day.
(e) 24 Hour Fitness wants to administer a satisfaction survey to its current members. Using its
membership roster, the club randomly selects 40 club members and asks them about their
level of satisfaction with the club.

All Rights Reserved. 2020 Abdul, Atienza, et. al.


Lesson 4 63

4. Patricia categorized her spending for this month into four categories: Rent, Food, Fun, and Other.
The percents she spent in each category are pictured here. If she spent a total of PhP 26,000 this
month, how much did she spend on rent?

S
DM
5. You recorded the time in seconds it took for 8 participants to solve a puzzle. The times were:
15.2, 18.8, 19.3, 19.7, 20.2, 21.8, 22.1, 29.4.

(a) Calculate the mean and the median time it took for the 8 participants to solve a puzzle.
(b) Calculate the range and standard deviation of the time it took for the 8 participants to solve
the puzzle.

6. Make up three data sets with 5 numbers each that have:

(a) the same mean but different standard deviations.


P
(b) the same mean but different medians.
(c) the same median but different means.
PU

All Rights Reserved. 2020 Abdul, Atienza, et. al.


Lesson 5 64

Lesson 5: Linear Programming

Learning Outcomes
At the end of the lesson, students should be able to

1. demonstrate understanding of a linear programming model;

2. identify the different components of a linear programming problem;

3. develop a linear programming model that involves maximization and minimization;

S
4. demonstrate understanding of linear inequalities;

5. identify the different steps in solving linear inequalities in two variables;

DM
6. find a solution space to a system of linear inequalities in two variables;

7. identify the different steps in solving linear programming problem in two variables using
graphical method;

8. find an optimal solution of a linear programming problem using the concept of corner points.

5.1 Modeling with Linear Programming


P
This lesson will familiarize you to a particular technique in operations research that is very useful in our
daily lives, more specifically in decision making that involves optimization of scarce resources. You will
learn how to develop a linear programming model out a problem that involves optimization. Different
component of a linear programming problem will also be discussed in this lesson. Graphical and algebraic
PU

method of solutions to different types of linear programming models will also be discussed. Solutions
to linear programming problems with the use of Microsoft excel solver will also be introduced at the
succeeding lessons.

Many businesses in the Philippines have a common goal that is to maximize their profit while minimizing
their operation cost. With limited resources, obtaining such goal is possible with the use of proper
planning and integrating linear programming technique. Linear Programming is a mathematical method
in maximizing or minimizing linear functions subject to set of linear constraints. In business production,
the objective function is a linear function that either maximizes profit or minimizes cost that is subject
to a set of linear inequalities called linear constraints. These set of linear constraints can be viewed as
set of production requirements that is usually limited in quantity.

All Rights Reserved. 2020 Abdul, Atienza, et. al.


Lesson 5 65

Example 39 (The RAVLAM Company). RAVLAM Company produces two types of abaniko fan, small
and large, from three raw materials R1 , R2 and R3 . The following tables shows the materials used in
their production and the profit they earned for product:

amt. of raw materials


per piece of (in gms) max. daily availability (in gms.)
small size large size
Raw Material (R1 ) 80 70 5,000
Raw Material (R2 ) 100 150 9,000
Raw Material (R3 ) 175 250 13,000

S
Profit Per Price PhP 5.00 PhP 7.00

Based on their past sales, the total demand for small size abaniko fan should not exceed by 50 units.

DM
RAVLAM Company wants to determine the best combination of their product that will maximize their
daily profit.

This is a typical type of problem where linear programming can be used. To be able to help the
RAVLAM Company in their objective, first we need to develop the necessary linear programming model
of this problem. Basically, there are three components of a linear programming model. These are:

1. decision variables that we want to determine;

2. objective function that we want to maximize or minimize;


P
3. set of linear constraints that the solution must satisfy.

Using these three basic components of linear programming model, we can transform the above problem
PU

into a linear programming model.

The first step is to define the decision variables. Decision variables are usually what are asked in the
problem to obtain your objective or goal. In the above problem, RAVLAM, Company wants to maximize
their daily profit by knowing how many abaniko fan of each sizes (small and large) will they produced
daily. Thus the decision variables of the LP model are defined as:

x1 = daily production of small abaniko fan


x2 = daily production of large abaniko fan
After defining the decision variable we can now define the objective function. Objective function is a
linear function or equation that defines what we want to optimize (maximize or minimize). Since the
company wants to maximize their profit, we can use the profit contribution of each type of abaniko
fan as seen in the above table to form the objective function. Given that the profit per piece of small

All Rights Reserved. 2020 Abdul, Atienza, et. al.


Lesson 5 66

abaniko fan and large abaniko fan are Php 5.00 and Php7.00 respectively, then the objective function of
the model is:
Maximize: P = 5x1 + 7x2 ;

where P = total profit.

The next step is to develop the linear constraints of the model. These are restrictions in the raw materials
of production and daily demand of the product. Constraints are usually set of linear inequalities that
define the scarcity or abundance of particular requirements in production. In constructing each constraint,
it is very useful to be familiarizing in the given inequality.

S
0 1 0 1
usage of raw material A @ daily availability of A
@ ≤ :
by both abaniko fan each raw material

DM
Always remember that each requirement in the production of abaniko fan will define exactly one con-
straint. The daily usage of each type of abaniko fan as shown in the table can be written as:

Daily usage of raw material R1 by small abaniko fan = 80x1 :


Daily usage of raw material R1 by large abaniko fan = 70x2 :

These imply that the daily usage of raw material R1 by both abaniko fan sizes is 80x1 + 70x2 . In similar
manner we can get the second and third constraints of the LP model using raw material R2 and R3,
that is,
Raw Material R2 : 100x1 + 150x2 ≤ 9; 000
P
Raw Material R3 : 175x1 + 250x2 ≤ 13; 000
Another constraint of the problem can be found on the total daily demand of the abaniko fan. Since
the daily total demand of small size abaniko fan should not exceed by 50 units, therefore, we restrict
PU

that x1 ≤ 50. Now we can finally write the final linear programming model for the “RAVLAM Company
production”. The complete model is

Maximize P = 5x1 + 7x2


Subject to: 80x1 + 70x2 ≤ 5000
100x1 + 150x2 ≤ 9; 000
175x1 + 250x2 ≤ 13; 000
x1 ≤ 50
x1 ; x2 ≥ 0:

The last constraints where x1 ; x2 ≥ 0 is called the nonnegativity restrictions of the decision variable.
Since the company is producing a product this implies that there should be no negative values of the
decision variables. All values of the decision variables that satisfy each linear constraint is called feasible

All Rights Reserved. 2020 Abdul, Atienza, et. al.


Lesson 5 67

solution. From this feasible solution the objective is to find the best feasible solution that will satisfy
the objective function called the optimal solution.

Example 40 (The Diet Problem). Jackie is a basketball player who regularly monitors his diet so that
regular intake of calories, sugar carbohydrates and protein will satisfy his daily minimum nutritional
requirements. Every day he prepares three foods egg, rice and chicken. Each day he must consume
at least 600 calories, 100 grams of sugar, 283 grams of carbohydrates and 300 grams of protein. The
nutritional content per unit of each food is shown in the following table.

calories sugar carbohydrates protein unit price


Egg (1 piece) 72 1.1 0.4 7 PhP 7.00

S
Rice (1 cup) 204 0.08 44.08 4.2 PhP 10.00
Chicken (100 grams) 195 0 0 29.55 PhP 25.00

Solution. Define the decision variables:

Let
DM
Develop a linear programming model that satisfies the daily nutritional requirements of Jackie at the
minimum cost.

x1 = amount of egg to be consumed.


x2 = amount of rice to be consumed.
x3 = amount of chicken to be consumed.
P
Then the linear programming model is

Maximize: C = 7x1 + 10x2 + 25x3


Subject to:
PU

72x1 + 204x2 + 195x3 ≥ 600 (calorie constraint)


1:1x1 + 0:08x2 ≥ 100 (sugar constraint)
0:4x1 + 44:08x2 ≥ 283 (carbohydrate content)
7x1 + 4:2x2 + 29:55x3 ≥ 300 (Protein Constraint)
x1 ; x2 ; x3 ≥ 0 (nonnegativity)

All Rights Reserved. 2020 Abdul, Atienza, et. al.


Lesson 5 68

5.2 Solution Set of Systems of Linear Inequalities in Two Variables

Finding the solution of a linear programming model that contains two decision variables requires finding
all the feasible solutions of the constraints. Since the constraints of a linear programming model is a set
of linear inequalities, it is necessary to have a deep knowledge in solution of system of linear inequalities.

Definition 23
A system of linear inequalities in two variables x1 and x2 is a set of two linear inequalities of the
form 8
<a1 x1 + b1 x2 ≥ (or ≤) c1

S
>

:a2 x1 + b2 x2 ≥ (or ≤) c2 ;
>

where ai ; bi ; ci ∈ R, for i = 1; 2. The set S of all ordered pairs (x1 ; x2 ) satisfying both the

DM
inequalities is called the solution space or feasible region of the system.

To solve the system, we usually proceed using the graphing method:

Step 1. Replace ≥ and/or ≤ symbols in the system by = and graph the lines that corresponds to each
inequality.

Step 2. Determine the solution space of each linear inequality using feasibility test. Choose a point
that is not on the linear equation and substitute it to the linear inequality. If it satisfies the
P
linear inequality then the region where the point belongs is the solution space of that particular
linear inequality.
PU

Step 3. Determine the intersection of the solution space of the two linear inequalities. All the points
within this region are the solution space or feasible region of the system of linear inequalities.

Example 41. Determine the solution space of the following system of linear inequalities using the
graphing method. 8
<x
>
+ 2y ≤ 4
:3x
>
+ 2y ≤ 6

Step 1. Graph the lines corresponding to the equations x + 2y = 4 and 3x + 2y = 6.

(a) The intercepts of x + 2y = 4 are (4; 0) and (0; 2). (Red Line)

(b) The intercepts of 3x + 2y = 6 are (2; 0) and (0; 3). (Blue Line)

All Rights Reserved. 2020 Abdul, Atienza, et. al.


Lesson 5 69

−4 −2 2 4 6

−5

S
Step 2. Find the solution space of each linear inequalities by feasibility test. We may choose the point
of origin (0; 0) as test point since it does not lie in either inequalities.

DM
At (0; 0), x + 2y ≤ 4 is equivalent to 0 ≤ 4, which is true. Therefore, all the points in the same side of
the line as the origin are solutions to x + 2y ≤ 4.

At (0; 0), 3x + 2y ≤ 6 is equivalent to 0 ≤ 6, which is true. Therefore, every point in the same side of
the line as the origin are solutions to 3x + 2y ≤ 6.

Step 3. Determine the solution space of the system by getting the intersection of the solution space of
each linear inequality.
P
5
PU

−4 −2 2 4 6

−5

5.3 Graphical Solution for a Linear Programming Model


Linear programming model in two variables can be solved using graphical method. Although three
variables linear programming model can also be solved using this method, it is highly advised to use the
algebra method since we are dealing with solution space in three dimensions. The following steps can
be used to solve linear programming model in two variables.

All Rights Reserved. 2020 Abdul, Atienza, et. al.


Lesson 5 70

Step 1. Determine the feasible solution or solution space of the constraints using solution of system of
linear inequalities in two variables.

Step 2. Determine the corner points of the solution space by getting the point of intersection of each
pair of lines that defines it.

Step 3. Evaluate the objective functions using this corner points. The corner points the yields the
optimum value (maximum or minimum) is the optimal solution of the linear programming
model.

Theorem 2

S
In any linear programming model, the optimal solution (if it exists) can be found in one of the
corner points of the solution space or feasible region of the constraints.

DM
Hence, the optimal solutions tot he linear programming problem can be achieved at one among the
corner points.

Example 42 (Solution of the (RAVLAM Company) Problem).

Maximize P = 5x1 + 7x2


Subject to: 80x1 + 70x2 ≤ 5000
100x1 + 150x2 ≤ 9; 000
175x1 + 250x2 ≤ 13; 000
P
x1 ≤ 50
x1 ; x2 ≥ 0:

Step 1. Solution space or feasible region of the constraints. Using a graphing software, the following
PU

graph was obtained:

Step 2. Identify the corner points of the feasible region by getting the intersection points of each pair
of lines that defines it.

All Rights Reserved. 2020 Abdul, Atienza, et. al.


Lesson 5 71

S
: Evaluate the objective functions using this corner points. The corner points the yields the optimum
value (maximum or minimum) is the optimal solution of the linear programming model.

DM
Starting at the point of origin as the first corner point, the following are the other corner points as shown
in the graph above.

Vertex 1: At (0; 0), P = 5(0) + 7(0) = PhP 0:00

Vertex 2: At (50; 0), P = 5(50) + 7(0) = PhP 250:00

Vertex 3: At (50; 14:286), P = 5(50) + 7(14:286) = PhP 350:00

Vertex 4: At (43:871; 21:29), P = 5(43:871) + 7(21:29) = PhP 368:39 (Max)


P
Vertex 5: At (0; 52), P = 5(0) + 7(52) = PhP 364:00.
PU

Decision. The RAVLAM Company should produce 43:871 (≈ 44) units of small size abaniko fan and
21:29(≈ 29) units of large size abaniko fan to be able to maximize their profit at Php 368:385 (≈
Php 367:00).

All Rights Reserved. 2020 Abdul, Atienza, et. al.


Lesson 5 72

Assessment
1. Sketch the solution sets of the following systems of linear inequalities.

(a) x − y ≤ 10 (b) 3x − 4y ≥ 14
x +y ≤5 x +y ≤5

2. Solve the following linear programming problems.

(a) In a manufacturing process, the final product has a requirement that it must weigh exactly
150 kgs.The two raw materials used are A, with cost of Php 6 per unit and B, with a cost of

S
Php 12 per unit. At least 15 units of B and no more than 20 units of A must be used. Each
unit A weighs 4 kgs; each unit of b weighs 10 kgs. How much of each type of raw material
should be used for each unit of final product to minimize cost?

DM
(b) Charot Polar Products makes downhill and crosscountry skis. A pair of downhill skis requires
2 man-hours for cutting, 1 man-hour for shaping and 3 man-hours for finishing while a pair of
crosscountry skis requires 2 man-hours for cutting, 2 man-hours for shaping and 1 man-hour
for finishing. Each day the company has available 140 man-hours for cutting, 120 man-hours
for shaping and 150 man-hours for finishing. How many pairs of each type of ski should the
company manufacture each day in order to maximize profit if a pair of downhill skis yields a
profit of Php 10 and a pair of cross-country skis yields a profit of Php 8?
P
PU

All Rights Reserved. 2020 Abdul, Atienza, et. al.


Lesson 6 73

Lesson 6: Mathematics of Graphs

Learning Outcomes
At the end of the lesson, the students are able to

1. Define and illustrate graphs, paths, circuits, trees, complete graphs, connected graphs and
weighted graphs;

2. Apply algorithms in finding Euler circuits and Euler paths in connected graphs;

S
3. Exhibit the Travelling Salesman Problem and apply algorithms in solving it;

4. Apply the Kruskal’s Algorithm in finding a minimum spanning tree for a weighted graph

Overview DM
5. Exhibit and solve a graph coloring problem

Graph Theory is the mathematics that shows how a pair of objects coming from a particular collection
is abstractly related, and ultimately how to face-off all of these pairs so that a concrete problem may be
addressed. Moreover, objects that are paired by distance-relation, time-relation, cost-relation, etc., are
falling under solutions for the respective problems for the shortest way, the fastest time, the cheapest
amount of expenses, etc. Graph Theory, with aid of a friendly geometry and some well-guided algorithms,
P
will deal with all of these concerns.

6.1 Graph Concepts and Models


PU

Definition 24
Let V be a non-empty set, and E be any set of ordered pairs over V . The pair (V; E) is called a
graph. We denote a graph by G = (V; E). V is called the vertex set of G and its elements as
vertices, while E is called the edge set of G and its elements as edges.

Example 43. Let

V = {0; 1; 2; 3; 4; 5};
E1 = {(x; y ) | x and y are either both odd or both even};
E2 = {(x; y ) | x 6= y }; and
E3 = {(x; y ) | 0 < |x − y | ≤ 2}:

All Rights Reserved. 2020 Abdul, Atienza, et. al.


Lesson 6 74

Let
G1 = (V; E1 ) = {(0; 0); (0; 2); (0; 4); (1; 1); (1; 3); (1; 5); (2; 0); (2; 2); (2; 4);
(3; 1); (3; 3); (3; 5); (4; 0); (4; 2); (4; 4); (5; 1); (5; 3); (5; 5)};
G2 = (V; E2 ) = {(0; 1); (0; 2); (0; 3); (0; 4); (0; 5); (1; 0); (1; 2); (1; 3); (1; 4); (1; 5);
(2; 0); (2; 1); (2; 3); (2; 4); (2; 5); (3; 0); (3; 1); (3; 2); (3; 4); (3; 5);
(4; 0); (4; 1); (4; 2); (4; 3); (4; 5); (5; 0); (5; 1); (5; 2); (5; 3); (5; 4)}; and
G3 = (V; E3 ) = {(0; 1); (0; 2); (1; 0); (1; 2); (1; 3); (2; 0); (2; 1); (2; 3); (2; 4);
(3; 1); (3; 2); (3; 4); (3; 5); (4; 2); (4; 3); (4; 5); (5; 3); (5; 4)}
are examples of graphs. A graph can be represented diagrammatically by letting the vertices illustrated

S
as nodes and edges as arcs connecting the nodes. The following diagrams represent graphs G1 ; G2 and
G3 .

G1 = (V; E1 )
DM G2 = (V; E2 )

Note that in G1 , a vertex is adjacent to itself. Such edges are called loops.
G3 = (V; E3 )
P
Definition 25
Vertices are said to be adjacent if there is an edge that joins them. Edges are said to be adjacent
if they share a common vertex. The degree of a vertex is the number of edges at that vertex.
PU

Example 44. The table below summarizes the degree of each vertex of the previously defined graphs
G1 ; G2 and G3 .

Vertex G1 G2 G3
0 3 5 2
1 3 5 3
2 3 5 4
3 3 5 4
4 3 5 3
5 3 5 2

All Rights Reserved. 2020 Abdul, Atienza, et. al.


Lesson 6 75

Definition 26
Given a graph G = (V; E). A path in G is a sequence of vertices with no repeated edges. A
circuit in G is a path that starts and ends at the same vertex.

Example 45. Consider the graph illustrated below.

S
The following are some paths:

• A-B-E-D

• A-B-C-A-D-E

• A-B-C-B-E

• A-C-B-E-E-D
DM
The following are not paths:

• A-C-A-D-E

• A-B-C-B-A-D

• A-D-E-E-D

• A-B-C-B-E-E-D-A-C-B
The following are some cir-
cuits:

• A-B-C-A

• A-D-E-B-A

• A-C-B-E-D-A
P
Definition 27
A graph G is said to be connected if there is a path joining any two of its vertices. Otherwise,
it is said to be disconnected.
PU

Example 46.

This is a connected graph. Any two distinct This is a disconnected graph.


vertices are joined by a path. No path takes A to G.

All Rights Reserved. 2020 Abdul, Atienza, et. al.


Lesson 6 76

Moreover, we can visualize graphs thru the following models.

1. In 1736, the attention of mathematician Leonhard Euler was caught by a simple puzzle: "Is it
possible to walk around and cross all the seven bridges of the old town of Konigsberg exactly
once?"

Source: 2010 Encyclopedia Britannica

S
DM
Four small islands are connected by seven bridges as shown above in the left figure. Its graph
representation is shown above in the right figure.

2. Can you continuously trace this figure starting from any vertex such that you can only pass through
every edge and every vertex once without lifting your pen?
P
3. The table below shows the distances of towns Alpra, Betra, Gamra at Deltra from each other, in
km.
PU

Alpra Betra Gamra Deltra


Alpra * 11 13 14
Betra 11 * 12 13
Gamra 13 12 * 15
Deltra 14 13 15 *

If a courier service personnel aims to drop packages once in each town, what routing schedule can
be made so that he can travel the shortest distance?

4. Suppose you are tasked to color a blank map of Metro Manila in such a way that no two adjacent
cities will have the same color. What is the least number of colors you can use to complete this
task?

All Rights Reserved. 2020 Abdul, Atienza, et. al.


Lesson 6 77

6.2 Euler’s Theorems and Fleury’s Algorithms

Definition 28
Given a connected graph G. An edge in G is said to be a bridge if G becomes disconnected when
it is deleted. An Euler path is a path that travels through every edge of G. An Euler circuit is
a circuit that travels through every edge of G.

Example 47.

S
DM
In this graph, paths E-A-B-C-A-D-G-F-D and
E-A-C-B-A-D-G-F-D are Euler paths.

Theorem 3: Euler’s Theorem 1


In this graph, paths A-B-C-A-D-F-G-D-E-A and
D-A-C-B-A-E-D-G-F-D are Euler circuits.

1. If a graph has any vertices of odd degree, then it cannot have an Euler circuit.
P
2. If a graph is connected and every vertex has an even degree, then it has at least one Euler
circuit.
PU

Example 48.

1. Let G1 be the graph shown below.

Vertices D and E have odd degrees. Hence, by Euler’s Theorem 1 (A), G1 has no Euler circuit.

2. Let G2 be the graph shown below.

All Rights Reserved. 2020 Abdul, Atienza, et. al.


Lesson 6 78

Every vertex of G2 has an even degree. By Euler’s Theorem 1 (B), G2 has at least one Euler circuit.

3. Let G3 be the graph shown below.

S
DM
Every vertex of G3 has an odd degree. Thus, Euler’s Theorem 1 (A) says G3 has no Euler circuit.

Theorem 4: Euler’s Theorem 2


1. If a graph has more than two vertices of odd degree, then it cannot have an Euler path.
P
2. If a graph is connected and has just two vertices of odd degree, then it has at least one Euler
path. Any such path must start at one of the odd-degree vertices and end at the other one.
PU

Example 49. 1. Let G1 be the graph shown below.

Only vertices D and E have odd degrees. Hence, by Euler’s Theorem 2 (B), G1 has at least one
Euler path. The path E-A-B-C-A-D-G-F-D is an Euler path.

2. Let G2 be the graph shown below.

All Rights Reserved. 2020 Abdul, Atienza, et. al.


Lesson 6 79

By Euler’s Theorem 1 (B), G2 has at least one Euler circuit. Thus, it has an Euler path.

3. Let G3 be the graph shown below.

S
Theorem 5: Euler’s Theorem 3
DM
Every vertex has an odd degree. Hence, by Euler’s Theorem 2 (A), G3 has no Euler path.

1. The sum of the degrees of all the vertices of a graph equals twice the number of edges.

2. The number of vertices of odd degree must be even.


P
Example 50. The table below characterizes the previously defined graphs G1 ; G2 and G3 of Example 7
in reference to Euler’s Theorem 3.
PU

Number of Vertices Number of Vertices Sum of the Degrees Number of


with Odd Degree with Even Degree of all Vertices Edges
G1 2 5 16 8
G2 0 7 18 9
G3 4 0 14 7

Given a connected graph with all vertices have even degrees, we can find an Euler circuit using Fleury’s
Algorithm.

All Rights Reserved. 2020 Abdul, Atienza, et. al.


Lesson 6 80

Fleury’s Algorithm for Finding Eulerian Circuit (Don’t cross the bridge until you have to).

1. Make sure that the graph is connected and all vertices have even degree.

2. Start at any vertex.

3. Travel through an edge if:

(a) it is not a bridge for the untraveled part, or


(b) there is no other alternative.

S
4. Label the edges in the order in which you travel them.

5. When you can’t travel any more, stop.

DM
Example 51. Using Fleury’s Algorithm, find an Euler Circuit in this graph.
P
Solution. START: The graph is connected. Every vertex has an even degree.

1. Choose vertex D, primarily due to having to having the highest degree.


PU

2. None of the edges (D,A), (D,E), (D, F) and (D, G) is a bridge. Hence, you can travel on any of
these edges. Say, take (D,A).

3. Label path D-A.

4. Omitting (D, A) in the graph, standing on A makes edge (A, E) the only bridge. Then, you can
travel on either (A, B) or (A, C). Say, take (A, C).

5. Label path D-A-C.

6. Standing on C, (C, B) is a bridge. Since there are no other edges adjacent to C, take (C, B).

7. Label path D-A-C-B.

8. Standing on B, (B, A) is a bridge. Since there are no other edge adjacent to B, take (B, A).

All Rights Reserved. 2020 Abdul, Atienza, et. al.


Lesson 6 81

9. Label path D-A-C-B-A.

10. Standing on A, (A, E) is a bridge. Since there are no other edges adjacent to A, take (A, E).

11. Label path D-A-C-B-A-E.

12. Standing on E, (E, D) is a bridge. Since there are no other edges adjacent to E, take (E, D).

13. Label path D-A-C-B-A-E-D.

14. Standing on D, neither (D,E) nor (D,G) is a bridge. Take (D, G).

S
15. Label path D-A-C-B-A-E-D-G.

16. Standing on G, (G, F) is a bridge. Since there are no other edges adjacent to F, take (G, F).

17. Label path D-A-C-B-A-E-D-G-F.

take (F, D).

19. Label path D-A-C-B-A-E-D-G-F-D.


DM
18. Standing on F, (F, D) is a bridge. Since it is the only remaining edge of the graph adjacent to F,

The path D-A-C-B-A-E-D-G-F-D is an Euler circuit.

Also, given a connected graph with exactly two vertices of odd degrees, we can find an Euler path using
P
Fleury’s Algorithm.

Fleury’s Algorithm for Finding an Eulerian Path (Don’t cross the bridge until you have to).
PU

1. Make sure that the graph is connected and only two vertices have odd degree.

2. Start at any of the two odd-degree vertices.

3. Travel through an edge if:

(a) it is not a bridge for the untraveled part, or


(b) there is no other alternative.

4. Label the edges in the order in which you travel them.

5. When you can’t travel any more, stop.

All Rights Reserved. 2020 Abdul, Atienza, et. al.


Lesson 6 82

Example 52. Using Fleury’s Algorithm, find an Euler Path in this graph.

START: The graph is connected. Only D and E are the vertices with odd degrees.

S
1. Choose vertex D, primarily due to having to having the highest degree.

3. Label path D-G. DM


2. Edge (D, A) is a bridge. Its omission makes the graph disconnected. Hence, you can travel on
either (D, F) and (D, G). Say, take (D,G).

4. Omitting (D, G) in the graph, standing on G makes edge (G, F) a bridge, yet the only edge to F.
Then, take (G, F).

5. Label path D-G-F.

6. Standing on F, (F, D) is a bridge. Since there are no other edges adjacent to F, take (F, D).
P
7. Label path D-G-F-D.
PU

8. Standing on D, (D, A) is a bridge. Since there are no other edges adjacent to D, take (D, A).

9. Label path D-G-F-D-A.

10. Standing on A, (A, E) is a bridge. Hence, you can travel on either (A, B) or (A, C). Say, take (A,
C).

11. Label path D-G-F-D-A-C.

12. Standing on C, (C, B) is a bridge. Since there are no other edges adjacent to B, take (C, B).

13. We have now path D-G-F-D-A-C-B.

14. Standing on B, (B, A) is a bridge. Since there are no other edges adjacent to B, take (B, A).

15. Label path D-G-F-D-A-C-B-A.

All Rights Reserved. 2020 Abdul, Atienza, et. al.


Lesson 6 83

16. Standing on A, (A, E) is a bridge. Since it is the only remaining edge of the graph adjacent to F,
take (A, E).

17. Label path D-G-F-D-A-C-B-A-E.

The path D-G-F-D-A-C-B-A-E is an Euler Path.

6.3 Hamilton Circuits, Hamilton Paths and the


Traveling-Salesman Problems

S
Definition 29
Given a connected graph G. A Hamilton circuit is a circuit that passes through each vertex

Example 53.

DM
exactly once. A Hamilton path is a path that passes through each vertex exactly once.

1. Let G1 be the graph shown below.

The path A-B-C-D-E is a Hamilton path.


P
The path A-B-C-D-E-A is a Hamilton circuit.

2. Let G2 be the graph shown below.


PU

The path A-B-C-G-D-E-F is a Hamilton path.


The path A-B-F-E-D-G-C-A is a Hamilton circuit.

3. Let G3 be the graph shown below.

All Rights Reserved. 2020 Abdul, Atienza, et. al.


Lesson 6 84

The path B-E-A-D-C is a Hamilton path.


G3 has no Hamilton circuits.

4. Let G4 be the graph shown below.

S
Definition 30 DM
G4 has neither Hamilton paths nor Hamilton circuits.

A graph Kn with n vertices is said to be a complete graph if every vertex is adjacent to the
other (n − 1) vertices.

Example 54. The following are examples of complete graphs.


P
PU

K1 K2 K3 K4

Definition 31
A weighted graph is a graph whose edges have assigned numbers. Such numbers are called
weights. Common weights are time, distance and cost. Complete graphs that are weighted are
simply called complete weighted graphs.

Example 55. Consider this complete weighted graph.

All Rights Reserved. 2020 Abdul, Atienza, et. al.


Lesson 6 85

S
The vertices represent 5 destinations and the weights represent the cost of reaching one destination from
another destination. Starting at any destination, what is the optimal solution so that all destinations

The Travelling Salesman Problem.


DM
will be reached exactly once and returns to its starting point? This problem is an example of a Traveling
Salesman Problem.

Suppose that a salesman has to visit each of a number of cities exactly once before returning to
his starting point. What is the shortest available route through the cities?

Remarks.
P
1. Given a complete weighted graph, TSP asks for the circuit of minimum total weight in a graph
that visits each vertex exactly once and returns to its starting point.

2. The optimal solution for a TSP is a Hamilton circuit for a complete weighted graph for which the
PU

sum of the weights of the edges traversed is the smallest possible number.

We have three methods to determine a solution for a TSP: the Brute Force Method, the Nearest Neigh-
bor Method and the Cheapest Link Algorithm.

THE BRUTE FORCE METHOD

1. Draw a complete weighted graph for the problem.

2. List all possible Hamilton circuits.

3. Find the sum of the weights of the edges for each circuit.

The circuit with the smallest sum is the optimal solution.

All Rights Reserved. 2020 Abdul, Atienza, et. al.


Lesson 6 86

Example 56. Using the Brute Force Method in this complete weighted graph,

S
DM
we obtain all of the following Hamilton circuits starting at A. Note that you can start at any point. Due
to the cyclic nature, you may find a Hamilton circuit starting at any point that is essentially the same
with one of the following.

Hamilton Circuits Sum of Weights


A-B-C-D-E-A 200 + 310 + 425 + 375 + 240 = 1 550
P
A-B-D-C-E-A 200 + 510 + 425 + 410 + 240 = 1 785
A-B-C-E-D-A 200 + 310 + 410 + 375 + 390 = 1 685
A-D-C-B-E-A 390 + 425 + 310 + 350 + 240 = 1 715
PU

A-B-E-D-C-A 200 + 350 + 375 + 425 + 415 = 1 765


A-C-B-D-E-A 415 + 310 + 510 + 375 + 240 = 1 850
A-C-B-E-D-A 415 + 310 + 350 + 375 + 390 = 1 840
A-C-D-B-E-A 415 + 425 + 510 + 350 + 240 = 1 940
A-C-E-D-B-A 415 + 410 + 375 + 510 + 200 = 1 910
A-D-B-C-E-A 390 + 510 + 310 + 410 + 240 = 1 860
A-B-E-C-D-A 200 + 350 + 410 + 425 + 390 = 1 775

The least cost is 1 550 and belongs to the Hamilton circuit A-B-C-D-E-A.

All Rights Reserved. 2020 Abdul, Atienza, et. al.


Lesson 6 87

THE NEAREST NEIGHBOR METHOD (AN APPROXIMATE SOLUTION)

1. Draw a complete weighted graph for the problem.

2. Starting at a designated vertex, pick the edge with the smallest weight and move to the
second vertex.

3. At the next vertex, pick the edge with smallest weight that doesn?t go to a vertex already
used.

4. Continue until the circuit is completed.

S
The sum of the weights is an approximation to the optimal solution.

DM
Example 57. Using the Nearest Neighbor Method, we determine an approximate solution in the given
complete weighted graph,
P
PU

1. Start at vertex A. The edge with the least cost adjacent with A is (A, B). Take (A, B) and stand
at B.

2. At vertex B, the edge with the least cost that is adjacent with B is (B, C). Take (B, C) and stand
at C.

3. At vertex C, the edge with the least cost that is adjacent with C is (C, A) However, it leads back
to A, so, disregard (C, A). The next edge with the least cost is (C, E). Take (C, E) and stand at
E.

4. At vertex E, the edge with the least cost that is not leading back to A, B and C, is edge (E, D).
Take (E, D) and stand at D.

5. Since all five vertices are accounted, take edge (D, A) to form a Hamilton circuit.

All Rights Reserved. 2020 Abdul, Atienza, et. al.


Lesson 6 88

S
DM
The Hamilton circuit formed is A-B-C-E-D-A having the cost 1,685. This method promises an approxi-
mate solution since results may vary depending on the starting point.
P
THE CHEAPEST LINK ALGORITHM

1. Draw a complete weighted graph for the problem.


PU

2. Pick the edge with the smallest overall weight. In case of a tie, pick at random.

3. Pick the edge with the next smallest overall weight that doesn’t:

(a) enclose a smaller circuit that doesn’t reach every vertex or


(b) result in three chosen edges coming from the same vertex.

4. Repeat Step 2 until the Hamilton circuit is complete.

Example 58. Using the Cheapest Link Algorithm, we determine an approximate solution in the given
complete weighted graph,

All Rights Reserved. 2020 Abdul, Atienza, et. al.


Lesson 6 89

S
1. The edge with the smallest overall weight is (A, B) with weight 200. Pick (A, B).

DM
2. Among the remaining edges, the edge with the smallest overall weight is (A, E) with weight 240.
Pick (A, E).

3. Among the remaining edges, the edge with the smallest overall weight is (B, C) with weight 310.
Pick (B, C).

4. Among the remaining edges, the edge with the smallest overall weight is (B, E) with weight 350.
But, (B, E) encloses a smaller circuit with (A, B) and (A, E). Reject (B, E).

5. Among the remaining edges, the edge with the smallest overall weight is (D, E) with weight 375.
P
Since, it doesn’t enclose a smaller circuit or result in three chosen edges coming from the same
vertex, then (D, E) can be picked.

6. Among the remaining edges, the edge with the smallest overall weight is (A, D) with weight 390.
PU

But, (A, D) encloses a smaller circuit with (A, E) and (D, E), and is the third edge adjacent with
A. Thus, reject (A, D).

7. Among the remaining edges, the edge with the smallest overall weight is (C, E) with weight 410.
But, (C, E) encloses a smaller circuit with (A, B), (B, C) and (A, E), and is the third edge adjacent
with E. Thus, reject (A, D).

8. Among the remaining edges, the edge with the smallest overall weight is (A, C) with weight 415.
But, (A, C) encloses a smaller circuit with (A, B) and (B, C), and will be a third edge adjacent
with A. Thus, reject (A, C).

9. Among the remaining edges, the edge with the smallest overall weight is (C, D) with weight 425.
Since, it doesn’t enclose a smaller circuit or result in three chosen edges coming from the same
vertex, then (C, D) can be picked.

All Rights Reserved. 2020 Abdul, Atienza, et. al.


Lesson 6 90

S
With the five vertices already chosen, we have the Hamilton circuit A-B-C-D-E-A and the optimal cost
as 1,550.

6.4
DM
Spanning Trees and Kruskal’s Algorithm

Definition 32
A tree is a graph in which any two vertices are connected by exactly one path.

The following are some properties of trees:

1. A tree has no circuits.


P
2. Trees are connected graphs.

3. Every edge in a tree is a bridge.


PU

4. A tree with n vertices has exactly (n − 1) edges.

Example 59. The following are examples of trees on 6 vertices.

All Rights Reserved. 2020 Abdul, Atienza, et. al.


Lesson 6 91

Definition 33
A spanning tree for a connected graph G of n vertices is a connected subgraph that is a tree on
n vertices.
A minimum spanning tree for a weighted graph is the spanning tree for that graph that has
the smallest possible sum of the weights.

A spanning tree for a graph is a tree that results from the removal of as many edges as possible from
the original graph without making it disconnected. To find the underlying minimum spanning tree in a
connected graph, one may apply the so-called Kruskal’s algorithm.

S
KRUSKAL’S ALGORITHM
To construct a minimum spanning tree for a weighted graph:

DM
1. Choose the edge with the lowest weight (and highlight it in color). If there is more than
one, pick one at random.

2. Choose the unmarked edge with the next lowest weight that does not form a circuit with
the edges already highlighted, (and highlight it).

3. Repeat until all vertices have been connected.

Example 60. Determine the minimum spanning tree in this weighted graph.
P
PU

Solution.

1. The edge with the lowest weight is (D; E) with weight 14. Pick (D; E).

2. Among the remaining edges, the edge with the lowest weight that does not form a circuit with
(D; E) is (D; F ) with weight 18. Thus, pick (D; F ).

3. Among the remaining edges, the edge with the lowest weight that does not form a circuit with
(D; E) and (D; F ) is (B; C) with weight 20. Thus, pick (B; C).

All Rights Reserved. 2020 Abdul, Atienza, et. al.


Lesson 6 92

4. Among the remaining edges, the edge with the lowest weight that does not form a circuit with
(D; E); (D; F ) and (B; C) is (B; F ) with weight 21. Thus, pick (B; F ).

5. Among the remaining edges, the edge with the lowest weight is (C; F ) with weight 24. But, (C; F )
forms a circuit with (B; C) and (B; F ). Thus, reject (C; F ).

6. Among the remaining edges, the edge with the lowest weight that does not form a circuit with
(D; E); (D; F ); (B; C) and (B; F ) is (A; B) with weight 27. Thus, pick (A; B).

With the six vertices already chosen, we have the required minimum spanning tree highlighted:

S
DM
6.5 Graph Coloring
P
Definition 34
Graph coloring is a function that assigns either the vertices or edges of a graph by a unique color
(or label). Graph coloring is specified as either vertex coloring or edge coloring depending
PU

whether the vertices or the edges are labelled.

Definition 35
A graph is said to be a planar graph if it can be drawn in a plane without the edges crossing.

Example 61. The graph

All Rights Reserved. 2020 Abdul, Atienza, et. al.


Lesson 6 93

is a planar graph since it be drawn in a plane without the edges crossing as

S
Definition 36
DM
Given a graph coloring problem, the smallest number of colors needed to color a graph is called
the chromatic number.
P
Observe that graphs representing the adjacency of countries, provinces or cities are planar graphs. The
most famous coloring problem is derived from the idea of coloring maps such that no two adjacent
countries, provinces or cities are of the same color. With the way of this coloring, its chromatic number
PU

can be determined.

Theorem 6: The Four Color Theorem


Every possible geographical map can be colored with at most four colors in such a way that no
two adjacent regions have the same color.

Two regions are called adjacent if they share a border segment, not just a point. To determine the
chromatic number for a given geographical map, it should be represented as a graph where a vertex
represents a region, and vertices are adjacent if the region they represent are adjacent in the map.

The following algorithm lets the fulfillment of the Four-Color Theorem.

All Rights Reserved. 2020 Abdul, Atienza, et. al.


Lesson 6 94

VERTEX COLORING ALGORITHM

1. Determine the vertex with the highest degree. Assign to it the first color.

2. Also, assign the first color to all vertices that are not adjacent to the first chosen vertex.

3. Among the remaining vertices, determine the vertex with the highest degree. Assign to it
the second color.

4. Also, assign the second color to all vertices that are neither adjacent to the second chosen
vertex nor to vertices that already received the second color.

S
5. Continue doing Steps 3 and 4 to until all vertices are colored.

The number of colors used is the chromatic number.

DM
Example 62. What is the chromatic number of the planar graph representing this map?
P
PU

The map show above has the its graph adjacency representation as

All Rights Reserved. 2020 Abdul, Atienza, et. al.


Lesson 6 95

Applying the Vertex Coloring Algorithm on the graph, we have the following.

1. The vertex with the highest degree is F with degree 7. Assign to F the first color (to be presented
as red).

2. The vertices A, C, D, L, M, N and O are not adjacent to A. Note that among these vertices, there
are also adjacent vertices. Choose the vertices strategically such that most of them will receiver
the first color. Say, choose vertices A, D, L, and N to receive the first color.

3. Among the remaining vertices, the vertices with the highest degree with noncolored vertices are B
and I. Choose one of them to receive the second color (to be represented by yellow). Say, B will

S
receive the second color.

4. The vertices H, I, J, K, M and O are not adjacent to B. Note that among these vertices, there are

DM
also adjacent vertices. Choose the vertices strategically such that most of them will receiver the
second color. Say, choose vertices H, J and M to receive the second color.

5. Among the remaining vertices, the vertex with the highest degree with noncolored vertices is G.
Hence, assign to G the third color (to be represented by green).

6. Vertices E, I and O are not adjacent to G. Also, none of them are adjacent. Thus, E, I and O will
receive the third color.

7. Lastly, vertices C and K are not adjacent. Hence, C and K will receive the fourth color (to be
represented as pink).
P
The following are the corresponding colored graphs and maps based on the algorithm.
PU

All Rights Reserved. 2020 Abdul, Atienza, et. al.


Lesson 6 96

Assessment
1. Let V = {cities of Metro Manila} and E = {(x; y ) | x and y are adjacent cities in Metro Manila}.

(a) Draw the graph G defined by G = (V; E). You may use initials to name a vertex representing
a city.

(b) Apply the Four-Color Theorem to determine the chromatic number of the vertex coloring for
G.

S
2. Apply Euler’s Theorems and Fleury’s Algorithm to determine Euler path and Euler circuits in each
graph.

(a)

DM (b)
P
3. A business man has to visit five cities A, B, C, D and E. The distance (in hundred miles) between
the five cities is as follows:
PU

A B C D E
A * 7 6 8 4
B 7 * 8 5 6
C 6 8 * 9 7
D 8 5 9 * 8
E 4 6 7 8 *

If the businessman starts from city A and has to come back to his starting point, which route
should he select so that the total distance travelled is minimum.

4. Apply Kruskal’s Algorithm to determine a minimum spanning tree in each graph.

All Rights Reserved. 2020 Abdul, Atienza, et. al.


Lesson 6 97

(a) (b)

S
DM
P
PU

All Rights Reserved. 2020 Abdul, Atienza, et. al.


Final Exam in GEED 10053

Instructions. This is a multiple choice exam. Fully shade the circle corresponding to your answer. If your answer
is not among the choices, shade the circle corresponding to choice E. Avoid erasures and do not put unnecessary
markings on the answer sheet. Turn to the next page for the problems.
Final Exam in GEED 10053 (Mathematics in the Modern World) Page 1

1. Which of the following best describes the true nature of mathematics?


(a) Mathematics consists of doing arithmetic operations and calculations.
(b) Mathematics sheds light to the nature of reality.
(c) Mathematics is the study of patterns and relationships.
(d) Mathematics is the study of the universe.

2. By ignoring the colors, which of the following logos is a cyclic rosette pattern?

(a) (b) (c) (d)

3. If a frieze pattern only admits symmetries generated by one translation, one horizontal reflection, and one glide
reflection, which of the following classifies the pattern using the John B. Conway naming system?
(a) Jump (b) Hop (c) Siddle (d) Step

4. Which of the following equals the 10th Fibonacci number F10 ?


(a) 21 (b) 34 (c) 55 (d) 89

5. Leonardo Da Vinci named this ratio as the “divine proportion”. This ratio can be approximated by taking a
large positive integer n and divide Fn by Fn+1 . What is the name of this ratio?
(a) Holy Ratio (b) Bronze Ratio (c) Silver Ratio (d) Golden Ratio

6. If p and q are given true propositions, which of the following is also true?
(a) p −→ (¬ q) (b) (¬ p) ∨ (¬ q) (c) p ∧ (¬ q) (d) (¬ p) −→ (¬ q)

7. Consider the conditional statement “If a pentagon has less than five sides, then an icosahedron has at least
fifteen faces.” Which of the following gives the contrapositive of the conditional statement?
(a) An icosahedron has at least fifteen faces only if a pentagon has less than five sides.
(b) If an icosahedron has less than fifteen faces, then a pentagon has at least five sides.
(c) If an icosahedron has at least fifteen faces, then a pentagon has less than five sides.
(d) If a pentagon has at least five sides, then an icosahedron has less than fifteen faces.

8. Which of the following collections is well-defined?


(a) the collection of all stars in the universe
(b) the collection of all real numbers whose square is irrational
(c) the collection of all sets
(d) the collection of all mathematicians 12 feet tall

9. Which of the following sets is equivalent to the set A = {4; 5; 6; 7; 8}?


(a) {1; 2; 3; 4} (b) {x | x is prime and 1 < x < 12}
(c) {x ∈ R | 4 ≤ x ≤ 8} (d) {n ∈ N | n < 9}
Final Exam in GEED 10053 (Mathematics in the Modern World) Page 2

10. In a class of 50 students, sixty percent use neither an iPhone nor an iPad. Twenty percent use an iPhone, while
thirty percent use an iPad. How many students in this class are using both an iPhone and an iPad?
(a) 5 (b) 10 (c) 15 (d) 30

11. Which of the following is a type of reasoning where we make conjectures based on observable examples?
(a) inductive reasoning (b) deductive reasoning (c) hyperactive reasoning (d) selective reasoning

12. The conjecture “If n is a positive integer, then n2 + n + 1 is prime” is false. Which of the following values of n
is a counterexample?
(a) n = 3 (b) n = 5 (c) n = 2 (d) n = 4

13. Which of the following illustrates deductive reasoning?


(a) I’m going to be rich someday because everyone in my family who graduates in college got rich and I just
graduated college.
(b) On Christmas day, movie theaters and Chinese restaurants are always open. Therefore, this Christmas, we
can go to a movie and get some Chinese take out food.
(c) My teacher usually give surprise quizzes on Friday. Today is Thursday, so I must review my lessons because
my teacher might give a surprise quiz tomorrow.
(d) Note that 92 = 81, 992 = 9; 801, and 9992 = 998; 001. Therefore, 99992 = 99; 980; 001.

14. Using inductive reasoning, determine which of the following is the sum of

1 1 1 1
+ + + ··· + ?
1·2 2·3 3·4 99 · 100

49 50 99 100
(a) 50
(b) 51
(c) 100
(d) 101

15. A piece of rope is 48 inches long and is cut so that one piece is twice the as long as the other. Which of the
following is the length of the longer piece?
(a) 8 in (b) 16 in (c) 24 in (d) 32 in

16. Which of the following is the type of sampling where every possible subsets of size n from a population of size
N has the same chance of being selected?
(a) purposive sampling (b) simple random sampling
(c) quota sampling (d) non-probability sampling

17. The totality of elements that we are interested to study in a statistical investigation is called the
(a) population (b) sample (c) parameter (d) statistic

18. This level of measure classifies data into categories which can be ranked; however, the precise differences of
these categories may not be clear. Which one is it?
(a) ratio level (b) interval level (c) ordinal level (d) nominal level
Final Exam in GEED 10053 (Mathematics in the Modern World) Page 3

19. Which of the following measures of central tendencies is most appropriate if the data can be measured up to
the nominal level only?
(a) mean (b) median
(c) mode (d) none of the other choices

20. This measure of variability can be calculated by taking the average deviation from the mean. Which one is it?
(a) range (b) variance (c) standard deviation (d) mean
For the next three items, consider the following scenario:
Chaka Furniture Shop located in Cavite makes two kinds of products, cabinets and dressers, which pass through
the assembly and the finishing departments. The Assembly department has 60 hours of work available each
week, while the finishing department has 48 hours each week. Making one cabinet needs 2 hours in Finishing
and 4 hours in Assembly, while it takes 2 hours to assemble a dresser and 4 hours to finish it. If the profit
generated per cabinet is P100 and per dresser is P80.(Hint: Let x be the number of cabinet produce and y be
the number of dresser produce).

21. Which of the following LP model is the best describe from the given problem?

(a) (b) (c) (d)


Max P = 100x + 80y Max P = 100x + 80y Max P = 80x + 100y Max P = 100x + 80y
subject to subject to subject to subject to
2x + 4y ≤ 60 4x + 2y ≤ 60 4x + 2y ≤ 60 4x + 2y ≥ 60
4x + 2y ≤ 48 2x + 4y ≤ 48 2x + 4y ≤ 48 2x + 4y ≥ 48
x; y ≥ 0 x; y ≥ 0 x; y ≥ 0 x; y ≥ 0

22. How many units of cabinet should the company produce to maximize the profit?
(a) 15 (b) 12 (c) 6 (d) 0

23. How many units of dresser should the company produce to maximize the profit?
(a) 12 (b) 9 (c) 6 (d) 0

(0; 8)

(5; 7)
24. Consider the objective function z = 2:5x + 1:5y . Which of the following is the
maximum value of z in the feasible region as shown in the right?
(6; 4)
(a) 7.5 (b) 12
(c) 23 (d) 25

(3; 0)

25. Which of the ff. shaded region will best describe the solution set of the system of inequalities:

(x ≥ 0) ∧ (y ≥ 0) ∧ (2x + y ≤ 4) ∧ (3x + 5y ≤ 15)?


Final Exam in GEED 10053 (Mathematics in the Modern World) Page 4

(a) (b) (c) (d)


26. How many edges are there in a graph with 10 vertices each of degree six?
(a) 10 (b) 20 (c) 30 (d) 40

27. What is the adjacency matrix that will represent the pseudograph as shown?

2 3 2 3 2 3 2 3
0 3 0 2 0 2 0 3 0 3 0 2 2 1 2 0
6 7 6 7 6 7 6 7
63 0 1 17 63 0 1 17 60 1 1 27 63 0 1 17
(a) 6
60
7 (b) 6 7 (c) 6 7 (d) 6 7
4 1 1 27
5
60
4 1 1 27
5
63
4 0 1 175
60
4 1 1 275
2 1 2 0 2 1 2 0 2 1 2 0 0 3 0 2
28. Which of the following graphs has neither a Hamilton circuit nor a Hamilton path?

(a) G1 only. (b) G2 only. (c) G3 only. (d) None of the above.

29. What is the height of the rooted tree shown below?

(a) 0 (b) 2 (c) 4 (d) 6

30. is a circuit that contains every edge of a graph exactly once.


(a) Euler Circuit (b) Euler Path (c) Hamiltonian Circuit (d) Hamiltonian Path

Das könnte Ihnen auch gefallen